Prc Board Questions in Pharma 2005

January 7, 2018 | Author: jose carlo araojo | Category: Antibiotics, Medical Treatments, Pharmacology, Drugs, Medical Specialties
Share Embed Donate


Short Description

pharma...

Description

BOARD QUESTIONS IN PHARMACOLOGY CHOOSE THE BEST ANSWER:

MPL = 44.5

1. A 40 year old female who underwent total abdominal hysterectomy with bilateral salpingo-oophorectomy. To prevent osteoporosis what HRT regimen will be best for her. She claims to have no history of breast cancer in the family: A. Estrogen only preparation B. Estrogen and medroxyprogesterone acetate C. Medroxyprogesterone acetate D. Raloxifene MPL- 0.25 REFERENCE: PHARMACOLOGY 4th Ed. Rang, pp. 440 2. The Patient at the delivery room is having uterine atony after giving birth to an 8-lb baby boy. Her BP at present is 150/90 mmHg. What agent should you give her: A. Methylergonovine maleate C. Carboprost B. Oxytocin D. Ergometrine MPL: 1 REFERENCE: PHARMACOLOGY 4th Ed. Rang, et al pp.450 3. An old female was treated for endometriosis for 6 months. She claims to have been given a drug with androgenic effects such as hirsutism deepening of the voice and acne. Which of the following drugs could have been given: A. GnRH agonist C. Danazol B. Combined oral contraceptive pills D. Medroxyprogesterone acetate MPL: 0.25 REFERENCE: PHARMACOLOGY 4th Ed. Rang, pp.447 4. A patient underwent hip replacement and is being given morphine for pain relief. At present her RR was noted to be at 8 cycles/min with prolonged episodes of apnea. What will you give in this patient? A. Atropine sulfate C. Naloxone B. Flumazenil D. Protamine sulfate MPL; 1 REFERENCE: PHARMACOLOGY 4th Ed. Rang, pp.598 5. If you have a patient with seizure disorder which anesthetic agent will not be appropriate: A. Enflurane C. Halothane B. Nitrous oxide D. Procaine MPL: 0.25 REFERENCE: PHARMACOLOGY 4th Ed. Rang, pp.523

6. A 50-year-old male with COPD is complaining of difficulty of sleeping for almost one month. Which of the following sedative hypnotics would be best for him: A. Diazepam C. Zolpidem B. Pentobarbital D. Thiopental MP: O.25 REFERENCE: PHARMACOLOGY 4th Ed. Rang, pp.534 7. On waking up a patient who just underwent knee surgery cannot remember what happened while he is at the Operating Room. The anxiolytic agent that was probably given to him was _______. A. Buspirone C. Hydroxyzine B. Lorazepam D. Thiopental MPL: 0.25 REFERENCE: PHARMACOLOGY 4th Ed. Rang, pp.536 8. A 22-year-old female was found unconscious in her room. She presently broke up with her boyfriend. By her bedside a number of sleeping pills and whiskey was noted. What could be the possible effect and interaction that occurred? A. Additive C. Potentiation B. Antagonism D. Synergism MPL: 0.33 9. If the patient will be given antidepressants. What adverse effect could be anticipated? A. Insomnia C. Sedation B. Diarrhea D.Tachycardia MPL: 0.33 REFERENCE: PHARMACOLOGY 4th Ed. Rang, pp.556 10. Antipsychotic potency generally runs parallel to the activity on which of the following receptors: A. D1 B. D2 C. D3 D. D4 MPL: 0.25 REFERENCE: PHARMACOLOGY 4th Ed. Rang, pp.487 Next page pls.

-

2–

11. The WHO classification of amphetamines: A. Cognition enhancer C. Neuroleptic B. Hallucinogen D. Psychostimulants MPL: 0.33 REFERENCE: PHARMACOLOGY 4th Ed. Rang, pp.469 12. If the patient claims to have visual hallucinations, what could be the possible psychotropic drug class that was taken? A. Antidepressant C. Psychotomimetic B. antipsychotic D. Psychomotor stimulant MPL: 0.25 REFERENCE: PHARMACOLOGY 4th Ed. Rang, pp.469 13. A student would like to improve her memory and cognitive performance. What class of psychotropic drug will she choose: A. Cognitive enhancers C. Psychostimulants B. Psychotomimetics D. Thymoleptics MPL: 1 REFERENCE: PHARMACOLOGY 4th Ed. Rang, pp.469 14. A bone cancer patient took high doses of morphine to relieve pain. What agent can be used to reverse its toxic effects? A. Nalbuphine C. Nalorphine B. Naloxone D. Methadone MPL: 1 REFERENCE: PHARMACOLOGY 4th Ed. Rang,pp.598 15. If respiratory depression would be the marked effect of the opioid analgesic, it is probably more selective to what receptor subtype? A. Delta C. Kappa B. Gamma D. Mu MPL: 0.25 REFERENCE: PHARMACOLOGY 4th Ed. Rang,pp.592 16. Which of the following is not a manifestation of a patient taking an opioid analgesic. Selectively interacting with mu receptors? A. Analgesia C. Physical dependence B. Dysphoria D. Respiratory depression MPL: 0.25 REFERENCE: PHARMACOLOGY 4th Ed. Rang, pp.592 17. If a child was brought to the clinic with skin abscess. Wound cotton done revealed the presence of Staph aureus. What antibiotic will be appropriate for this child:

A. Amoxicillin C. Cloxacillin B. Cephalexin D. Vancomycin MPL: 1 REFERENCE: PHARMACOLOGY 4th Ed. Rang, pp.693 18. The second generation cephalosporin which crosses the blood brain barrier: A. Cefoxime C. Cefoxitin B. Cefuroxime D. Cefoperazone MPL: 1 REFERENCE: PHARMACOLOGY 4th Ed. Rang, p.694 19. A G2P1 term, will be given prophylactic antibiotic prior to CS. Which of the following should be recommended: A. Cephalexin C. Cefazolin B. Cefoxitin D. Ceftriaxone MPL: 0.33 20. After prolonged treatment with penicillin, the patient developed pseudomembranous colitis. What antibiotic will you give for this case: A. Aztreonam C. Imipenem B. Ceftriaxone D. Vancomycin MPL: 0.25 REFERENCE: PHARMACOLOGY 4th Ed. Rang,p.702 21. This agent inhibits topoisomerase II (DNA gyrase) A. Clindamycin C. Spectinomycin B. Ciprofloxacin D. Tetracycline MPL: 0.25 REFERENCE: PHARMACOLOGY 4th Ed. Rang, p.720

Next page pls.

-

3–

22. In case of gram-negative septicemia if Ampicillin will be given with Gentamicin. What will be the expected response? A There will be better chances of resolution B B. Similar effect with monotherapy C. Disease resolution will be delayed D. No response will be noted MPL:1 REFERENCE: PHARMACOLOGY 4th Ed. Rang Pp. 698 23.. A neonate was given IV antibiotics for 3 days. However, the baby had hypothermia, diarrhea and other gray color. The baby was probably given: A. Amikacin C. Chloramphenicol B. Erythromycin D. Gentamycin MPL: 1 REFERENCE: PHARMACOLOGY 4th Ed. Rang,p.692 24. The following antibiotics inhibit protein synthesis. Which of the following inhibits the translocation process: A. Erythromycin C. Netilmicin B. Chloramphenicol D. Tetracycline MPL: 0.25 REFERENCE: PHARMACOLOGY 4th Ed. Rang,p.699 25. The purpose of giving compound drug therapy in the treatment of tuberculosis: A. to rapidly eradicate the strains of tubercle bacilli B. to shorten the infections phase C. to decrease the emergence of resistant organisms D. to prevent complications of tuberculosis MPL: 0.33 REFERENCE: PHARMACOLOGY 4th Ed. Rang,p.706 26. The patient on the 4th month of anti-TB treatment is having peripheral neuropathy. This may be attributed to which of the following: A. Isoniazid C. Pyrazinamide B. Ethambutol D. Streptomycin MPL: 1 REFERENCE: PHARMACOLOGY 4th Ed. Rang,p.704

27. Which of the following is NOT a first line agent in the treatment of tuberculosis? A. Ethambutol C. Rifampicin B. Pyrazinamide D. Streptomycin MPL: 1 REFERENCE:

PHARMACOLOGY

4th

Ed.

Rang,p.706

28. This agent is NOT included in the management if lepromatous leprosy: A. Capreomycin C. Dapsone B. Clofazimine D. Rifampicin MPL: 0.33 REFERENCE: PHARMACOLOGY 4th Ed. Rang,p.706 29. It is important for the clinician to avoid selection of an antiretroviral regimen that contains agents with similar toxicity profiles. Which regimen below contains at least 2 agents with similar toxicity profiles? A. Zidovudine, Lamivudine, Indinavir C. Zidovudine, Didanosine, Saquinavir B. Didanosine, Zalcitabine, Nelfinavir D. Stavudine, Lamivudine, Indinavir MPL: 0.25 REFERENCE: PHARMACOLOGY 4th Ed. Rang,p.714 30. A guanosine derivative which selectively inhibits viral DNA polymerase: A. Aciclovir C. Vidarabine B. Amantadine D. Zidovudine MPL: 0.25 REFERENCE: PHARMACOLOGY 4th Ed. Rang,p.713 31. A patient taking Warfarin was also being treated for fungal infection. After 7 days, the patient developed epistaxis, Which of the following antifungal agents may have this interaction with Warfarin? A. Nystatin C. Flucytosine B. Ketoconazole D. Amphotericin B MPL: O.25 REFERENCE: PHARMACOLOGY 4th Ed. Rang,p.317 Next page pls.

-

4–

32. If the patient became anemia with neutropenia and thrombocytopenia during antifungal treatment. . What is the possible agent she is taking? A. Griseofulvin C. Flucytosine B. Fluconazole D. Terbinafine MPL: 0.25 REFERENCE: PHARMACOLOGY 4th Ed. Rang,p.722 33. A7 year old patient was admitted because of pneumonia. On routine stool exam E. Histolytic Cyst 5 – 8/hpf was noted: A. Anti- amebic treatment not necessary B. Give diloxanide furoate C. Give metronidazole D. Give both diloxanide furoate and metronidazole MPL: 0.33 REFERENCE: PHARMACOLOGY 4th Ed. Rang,p.736 34. Which of the following anti-malarial agent may promote radical cure? A. Chloroquine C. Primaquine B. Mefloquine D. Pyrimethamine MPL: 0.25 REFERENCE: PHARMACOLOGY 4th Ed. Rang,p.728 35. The drug of choice for mixed round worm infection: A. Pyrantel pamoate C. Niclosamide B. Mebendazole D. Praziquantel MPL: 0.50 REFERENCE: PHARMACOLOGY 4th Ed. Rang,p.740 36. On follow-up, after treatment with tapeworm infection, scolex was noted in the stool of the child: A. Praziquantel C. Mebendazole

B. Niclosamide D. Ivermectin MPL: 0.25 REFERENCE: PHARMACOLOGY 4th Ed. Rang,p.742 37. Classified as anti-cancer antimetabolite agent: A. cyclophosphamide C. Methotrexate B. Doxorubicin D. Paclitaxel MPL: 0.25 REFERENCE: PHARMACOLOGY 4th Ed. Rang,p.676 38. An agent used for cancer chemotherapy that inhibits purine synthesis: A. Cytarabine C. Mercaptopurine B. Fluorouracil D. Vincristine MPL: 0.50 REFERENCE: PHARMACOLOGY 4th Ed. Rang,p.677 39. Does not cause myelosuppression: A. Bleomycin C. Etoposide B. Cisplatin D. Dactinomycin MPL: 0.25 REFERENCE: PHARMACOLOGY 4th Ed. Rang,p.678 40. In a prescription made by an internist, the generic name was enclosed in a parenthesis and written below the brand name. This will be interpreted by the drug store as: A. Violate prescription C. Erroneous prescription B. Impossible prescription D. Correct prescription MPL: 0.50 Generics Act of the Philippines 41. If the physician makes a prescription order that utilizes a drug supplied by the pharmaceutical company, what class of prescription order was made: A. Extemporaneous C. Precompounded B. Compounded D. Erroneous MPL: 0.25 42. The response of a patient who has bronchial asthma on the following drugs maybe graded as follows: A. Epinephrine > Norepinephrine > Isoproterenol B. Isoproterenol > Epinephrine >> Norepinephrine C. Isoproterenol > Epinephrine = Norepinephrine C D. Isoproterenol = Epinephrine >> Norepinephrine

MPL: 0.33 Rang,p.160

REFERENCE: PHARMACOLOGY 4th Ed.

Next page pls.

-543. A patient with septic shock was noted to have absent urine output for the past 12 hours. Which of the following drugs would be most helpful to this patient? A. Furosemide C. Dopamine B. Norepinephrine D. Epinephrine MPL: 0.33 REFERENCE: PHARMACOLOGY 4th Ed. Rang,p.288 44. A 40 year old male was seen at the ER because of anaphylaxis and was immediately given Epinephrine. Apparently, The patient has taken prazosin for his hypertension. Which of the following maybe observed in this patient? A. The patient may develop hypertensive crisis. a. B. There will be no effect on the patient’s blood pressure. C. The patient may develop hypotension. D D. The patient may develop severe difficulty of breathing due to bronchoconstriction. E MPL: 0.33 REFERENCE: PHARMACOLOGY 4th Ed. Rang,P.152

45. Which of the following actions of norepinephrine is blocked by prazosin? A. Bronchial smooth muscle relaxation B. Vascular smooth muscle contraction B. Renin release C. Increased heart rate MPL: 0.25 REFERENCE: PHARMACOLOGY 4th Ed. Rang,p.152 46. Bradycardia maybe noted as reflex response when a patient is given A. Dobutamine C. Norepinephrine B. Propranolol D. Methyldopa MPL: 0.25 REFERENCE: PHARMACOLOGY 4th Ed. Rang,p.127 47. Rifampicin and INH are given together for the treatment of pulmonary tuberculosis because their drug interaction is A. Additive C. Potentiation B. Synergism D. Antagonism MPL: 0.33 REFERENCE: PHARMACOLOGY 4th Ed. Rang,p.703 48. If the plasma concentration of a drug declines with first order kinetic, this means that A. there is only one metabolic path for drug disposal B. the rate of elimination is proportional to the remaining concentration b. C. the drug is largely metabolized in the liver after oral absorption and has low bioavailability D. the drug is not distributed outside the vascular systems MPL: 0.50 REFERENCE: PHARMACOLOGY 4th Ed. Rang,p.87 49. The following is NOT phase II drug metabolized reaction: A. Oxidation C. Methylation B. Glucoronidation D. Acetylation MPL: 0.25 REFERENCE: PHARMACOLOGY 4th Ed. Rang,p.79 50. Mr. Jose Vida was admitted due to severe pneumonia. He was given a drug with a volume of distribution of 40L and clearance of 80ml/min. What should be the loading dose in order to achieve a therapeutic plasma concentration of 40mg/L? A. 0.1 mg/L C. 115.2 mg

B. 10 mg D. 160 mg MPL: 0.25 REF. BASIC & CLINICAL PHARMACOLOGY 9th Ed. KATZUNGp.46 51. Two drugs A and B have the same mechanism of action. Drug A at a dose of 5 mg produce the same magnitude of effect as drug B at a dose of 500 mg. This means that A. Drug B is less efficacious than drug A B. Drug A is 100x more potent than drug B C. toxicity of drug A is less than that of drug B F D. Drug A is more effective than drug B. MPL: 0.50 REFERENCE: PHARMACOLOGY 4th Ed. Rang,p.29

Next page pls.

-6A 67 year old male was seen at the OPD due to epistaxis which occurred thrice for the past week and easy bruising. He also complained of abdominal discomfort and lack of appetite. PMH: Atrial fibrillation, Nocturnal heartburn Social history: Chronic alcohol abuse Medications: Warfarin, Digoxin, Cimetidine, Procainamide Pertinent laboratory exam: INR – 4.5

52. Which of the following could have contributed to the INR result of the patient? A. Cimetidine C. Chronic alcoholism B. Procainamide D. Digoxin MPL: 0.50 REFERENCE: PHARMACOLOGY 4th Ed. Rang,p.374 53. The INR result is brought about by: A. Increased thrombin activity B. Decreased platelet activation C. Diminished levels of vitamin K c. D. Decreased levels of factors II, VII. IX and X MPL: 1 REFERENCE: PHARMACOLOGY 4th Ed. Rang,p.316 54. In the event this patient develop massive bleeding, which of the following is the best thing to be done? A. Discontinue the drug and give vitamin K B. Discontinue the drug and change to LMWH. d. C. Discontinue the drug and give protamine sulfate e. D. Discontinue the drug and transfuse platelets. MPL: 0.33 REFERENCE: PHARMACOLOGY 4th Ed. Rang,p.319 55. If this patient’s ventricular rate cannot be controlled with digoxin, Which of the following maybe used as an alternative for ventricular rate control? A. Amlodipine C. Quinidine B. Atenolol D. Ibutilide MPL: 0.25 REFERENCE: PHARMACOLOGY 4th Ed. Rang,p.269 56. Which of the following is an adverse effect of procainamide? A. Hypertension C. drug induced Lupus erythematosus B. Bradycardia D. Corneal microdeposits MPL: 0.25 REFERENCE: PHARMACOLOGY 4th Ed. Rang p.229

A 26 year old medical student was brought to the ER for an apparent suicide poisoning due to broken heart. The maid could not recall the medication but claimed that the bottle contained medicine for headache. On PE, the physician noted cyanosis of the oral mucosa, tongue and nail beds. 57. Which of the following is the most likely drug ingested? A. Aspirin C. Flurbiprofen B. Acetaminophen D. Meloxicam MPL: 0.25 58. This DMARD agent is also an immunosuppressant. A. Gold C. Chloroquine B. Penicillamine D. Cyclosporine MPL: 0.50 REFERENCE: PHARMACOLOGY 4th Ed. Rang p.237 59. A 52 year old woman was suffering from severe joint pains and was diagnosed to have RA. However, she also appears to be suffering from acid peptic disease. Among the following, which is the safest to give her? A. Aspirin C. Celecoxib B. Ibuprofen D. Phenylbutazone MPL: 0.50 Next page pls.

-760. Which of the following NSAID has the longest half-life and therefore should be given at a longer dosage interval? A. meloxicam (20h) C. Tenoxicam (72h)

B. Rofecoxib (17h) MPL: 1

D. Piroxicam (50-60h) REFERENCE: PHARMACOLOGY 4th Ed. Rang p.230

A 21 year old male after quarrelling with his girlfriend took a bottle of insecticide from their garden approximately 30 –40 cc. He went back to his room and talked with his girlfriend over the phone. While talking to his girlfriend, he complained of dizziness, blurring of vision, generalized body weakness, numbness, severe epigastric pain and shortness of breath. He was able to call the attention of his older sister before falling on the floor. He was found lying on the floor and was noted to be pale, drowsy, with stiffening of the trunk and extremities and soaked with sweat. He was then brought to the ER. On arrival, he has drooling of saliva with 3 episodes of white viscid material, non-projectile non-bile stained about 2 tbsp. per bout. 61. As an ER physician, which of the following are you going to give the patient? A. Pilocarpine C. Atropine B. Physostigmine D. Carbachol MPL: 0.33 REFERENCE: PHARMACOLOGY 4th Ed. Rang p.122 62. A diabetic patient develops hyperlipedemia (elevated LDL, and triglycerides, normal HDL). Which of the following is NOT appropriate for the patient? A. Colestipol C. lovastatin B. Gemfibrozil D. niacin MPL: 0.25 REFERENCE: PHARMACOLOGY 4th Ed. Rang p.308 63 . The patient was noted to have an elevated triglycerides level after treatment with a hypolipidemic agent. This side effect could be secondary to which of the following? A. Cholestyramine C. cerivastatin B. Clofibrate D. niacin MPL: 0.25 REFERENCE: PHARMACOLOGY 4th Ed. Rang p.307 64. Inhibits de novo synthesis of cholesterol by inhibiting HMG CoA reductase: A. Atorvastatin C. gemfibrozil B. Colestipol D. niacin MPL: 0.50 REFERENCE: PHARMACOLOGY 4th Ed. Rang p.305

65. The patient has been having intermittent episodes of dry cough. The agent that may have probably caused this is ______________: A. Captopril C. losartan f. B. candesartan D. telmisartan MPL: 1 REFERENCE: PHARMACOLOGY 4th Ed. Rang p.292 66. A patient with tachyarrhythmia and hypertension would be given an antihypertensive. What agent would be the choice for this patient? A. Felodipine C. nifedipine B. isradipine D. verapamil MPL: 0.25 REFERENCE: PHARMACOLOGY 4th Ed. Rang p.275 67. -blockers are given with vasodilators due to the following reasons: A. reduce renal sodium excretion B. reduce renin release C. decrease systemic vascular resistance g. D. increase cardiac contractility MPL: 0.50 REFERENCE: PHARMACOLOGY 4th Ed. Rang p.287 68. Which of the following conditions will NOT benefit from the use of VASODILATORS: A. hypertension C. angina B. migraine D. peripheral vascular disease MPL: 0.50 REFERENCE: PHARMACOLOGY 4th Ed. Rang p.290 Next page pls.

-8-

69. .An asthmatic patient came to the ER because of generalized wheezing in the lungs. Urgent management was done. However after awhile hypertension was noted. Which of the following medications was probably given? A. salbutamol C. terbutaline B. isoproterenol D. salmeterol MPL: 0.50 REFERENCE: PHARMACOLOGY 4th Ed. Rang P.160 70. .A flight attendant has been having allergic rhinitis. She comes to your clinic asking for medication prescription 2 hours prior her to flight. What will be appropriate for her? A. diphenhydramine C. loratadine B. Chlorpheniramine D. promethazine MPL: 0.50 REFERENCE: PHARMACOLOGY 4th Ed. Rang p.345 71. It acts by inhibiting the immediate phase of asthma as it prevents mediator release from mast cells A. cromolyn sodium C. montelukast B. beclomethasone D. zileuton MPL: 0.33 REFERENCE: PHARMACOLOGY 4th Ed. Rang p.347 72. An asthmatic patient maintained on oral salbutamol for 8 months, apparently had infrequent asthma attacks for the first 6 months. However, at present she had increased frequency of asthma attacks. This could be secondary to: A. inadequate dose C. hypoxemia B. poor compliance D. tachyphylaxis MPL: 0.33 REFERENCE: PHARMACOLOGY 4th Ed. Rang p.16 73. Which of the following is NOT a pharmacologic action of Cortisone: A. anti-inflammatory C. promotes fetal lung maturation B. immunosuppressant D. adrenal suppression MPL: 0.33 REFERENCE: PHARMACOLOGY 4th Ed. Rang p.416 74. A patient with severe persistent asthma was treated for 6 months now with prednisone. Which of the following would be the expected side effect? A. Cushingoid facie C. hyperkalemia

B. hypoglycemia D. hypotension MPL: 0.50 REFERENCE: PHARMACOLOGY 4th Ed. Rang p.423 75. If a patient after prolonged steroid treatment developed depression. What synthetic corticosteroid agent is he taking? A. cortisone C. fludrocortisone B. dexamethasone D. prednisone MPL: 0.50 REFERENCE: PHARMACOLOGY 4th Ed. Rang p.552 FM, 40 year old male was seen at the clinic for check-up. He has bradykinesia, muscle rigidity, pill rolling movement, resting tremors and shuffling gait. 76. The single most effective agent in the treatment og this disorder is A. Levodopa C. Carbamazepine B. Baclofen D. Tacrine MPL: 1 REFERENCE: PHARMACOLOGY 4th Ed. Rang p.509 77. Which of the following is NOT TRUE about this drug? A. In clinical practice, it is almost always used in combination with peripherally acting inhibitor of aromatic L-amino acid decarboxylase. B. This drug has a short half-life about 1 – 3 hr. C. Administration of this drug with meals delay its absorption. h. D. This drug has very little side effect and can be withdrawn abruptly. MPL: 0.33 REFERENCE: PHARMACOLOGY 4th Ed. Rang p.510 78. Which of the following is a selective inhibitor of MAO-B A. Selegiline C. Tolcapone B. Amantadine D. Bromocriptine MPL; 0.50 REFERENCE: PHARMACOLOGY 4th Ed. Rang p.511

Next page pls.

-9GM, a 53 year old male was seen at the OPD for regular check-up. He had a history of type II DM for 5 years and hypothyroidism for 20 years. Medications: Levothyroxine, Glyburide 79.The patient had a series of laboratory examinations and was noted to have hyperlipedemia. Which of the following drugs can interfere with the absorption of levothyroxine? A. Simvastatin C. Gemfibrozil B. Cholestyramine D. Niacin MPL: 0.33 REFERENCE: PHARMACOLOGY 4th Ed. Rang p.70 LC, a 25 year old patient was seen at the clinic because of lump on her throat. She has gained 5 kg for the past 6 months, has cold intolerance, unusual painful heavy menses and constipation. She complained that the lump on her throat interferes with her swallowing. PMH: iron deficiency anemia of 4 months duration Medications: Ferrous sulfate, Ibuprofen, Kelp tablets(contains Iodine), Lo-ovral (Ethinyl estradiol and norgestrel) daily 80.What effect does the use of oral contraceptive have on the patient’s hypothyroidism? A. Low dose oral contraceptives are unlikely to affect her test results. B. Estrogen may falsely increase TSH due to alterations in TSH secretion C. Estrogen may falsely decrease FT4I due to alterations in TBG. D. Estrogen may falsely elevate total T4 levels due to alterations in TBG.

MPL: 0.33 REF. BASIC & CLINICAL PHARMACOLOGY 9th Ed. KATZUNGp.630 81. If LC becomes pregnant, how might her levothyroxine replacement be affected? A. She may require a 20-30% increase in dose. B. She may require a 20-30% decrease in dose. C. She will probably not require any changes in dose. D. TT4 levels should also be monitored because of changes in TBG. MPL: 0.33 REF. BASIC & CLINICAL PHARMACOLOGY 9th Ed. KATZUNG p.631 A 62 year old male was seen at the ER because of lightheadedness, palpitation and shortness of breath. He said that the palpitations were associated with exercise that usually went away with rest. Two days ago, while washing the dishes, he began to have shortness of breath and felt that his heart was “racing”. PMH: hypertension x 20 years; Hyperlipidemia for 5 years, and RHD with MVP as a child. Medications: Lisinopril, Furosemide, Gemfibrozil 82. Which of the following drugs is likely to be least effective in controlling this patient’s rapid ventricular response? A Digoxin C. Verapamil B. Diltiazem D. Atenolol MPL: 0.25 REFERENCE: PHARMACOLOGY 4th Ed. Rang p.265 83. This patient was initially given verapamil for his AF. Due to the addition of this drug, which of the following is the most appropriate treatment of this patient’s hypertension? A.Continue lisinopril and furosemide B. Continue lisinopril and discontinue furosemide C. Continue lisinopril D. Continue lisinopril, discontinue furosemide and add hydrochlorothiazide MPL: 0.25 REFERENCE: PHARMACOLOGY 4th Ed. Rang p.361 A 37 year old Fil-Am was seen at the clinic because of nausea and vomiting. Five days prior to consultation, he had nausea, vomiting, fever and chills. He took ibuprofen for generalized ache 3 days ago. Recent blood sugar reading was unavailable because he ran

out of test strips . On admission, he was weak looking, severely dehydrated with weak pulses. PMH: DM type I for 21 years, one episode of DKA 4 months ago; peripheral neuropathy Medications: Insulin NPH< Ibuprofen, Amitryptilline Next page pls.

- 10 84. Which of the following agents is least likely to aggravate the patient’s condition? A. Furosemide C. Acetaminophen B. Amitryptilline D. Insulin MPL: 0.33 REFERENCE: PHARMACOLOGY 4th Ed. Rang p.363 85. Loop diuretics have their principal diuretic effect on: A. collecting ducts B. ascending limb* C. distal convoluted tubules D. proximal convoluted tubules MPL: 0.25 REFERENCE: PHARMACOLOGY 4th Ed. Rang p.361 86. Which of the following are potential side effects of thiazide diuretics? A. hypokalemia, hyperglycemia, hyperlipidemia B. hypokalemia, ototoxicity, hyperuricemia C. hyperkalemia, alkalosis, nausea/vomiting D. hyperglycemia, hypokalemia, metabolic alkalosis* MPL: 0.33 REFERENCE: PHARMACOLOGY 4th Ed. Rang p.364

87. Patients receiving insulin therapy generally require which of the following interventions with progression to end-stage renal disease? A. Decrease in total insulin dose C. Increase in frequency of insulin administration B. Increase in total insulin dose D. No change in insulin regimen MPL: 0.25 REF. BASIC & CLINICAL PHARMACOLOGY 9th Ed. KATZUNG p.694 BK a 23 year old female seen at the ER because of nausea and vomiting. Myalgia, polydipsia and polyuria. Three days prior to consult, he attended a party and drank an excessive amount of alcohol. He woke up sick to his stomach, vomited 6x since then and was unable to eat nor drink. He stopped taking insulin and currently has headache. PMH: Type I DM for 11 years. Depression and allergic rhinitis Medications: Human insulin, Sertraline, Fluticasone, Loratidine, Acetaminophen

a.

B. blocking the release of Ach from nerve endings D. stimulating vagus REFERENCE: PHARMACOLOGY 4th Ed. Rang

MPL; 0.25 p.130 91. A 60 year old male underwent cataract removal. After surgery Ach chloride was administered intraocularly to: A. Relax the circular muscle of the iris C. decrease tearing from lachrymal secretion b. B. ensure complete miosis D. decrease the flow of aqueous humor MPL; 0.50 REFERENCE: PHARMACOLOGY 4th Ed. Rang p.118

Next page pls. 88. If Bk.’s blood glucose does not decrease by at least 2.8 mg/dl in the first 2 hours, what should be done? A. Double insulin infusion rate C. Increase the insulin infusion to 0.5 /kg/hr B. Give insulin bolus dose of 2U/kg D. Check blood glucose again in 2 hours MPL: 0.50 89. Which of the following statements about mixing NPH and regular insulins is NOT TRUE? A. The injection must be taken within 10 mins of mixing B. Regular insulin is drawn up before the NPH dose C. Mixtures of NPH and regular insulin are stable in any ratio D. Mixing insulins allows greater flexibility versus 70/30 insulin MPL: 0.25 90. Clostridium botulinum toxin produces respiratory paralysis by A. blocking nicotinic receptors C. causing circulatory collapse

- 11 92. An elderly patient was diagnosed to have open angle glaucoma. She was given 0.25% pilocarpine 2 gtts every 6 hours. The anticipated effect after the administration would be A. relax ciliary muscle B. improve accommodation C. relax sphincter muscle of iris D. contract ciliary muscle and pull on trabecular network to relieve pressure

MPL: 0.33 REFERENCE: PHARMACOLOGY 4th Ed. Rang p.121 93. Tachyphylaxis maybe observed with the use of the following: A. Propranolol C. Ephedrine B. Alpha methyl tyrosine D. Phenylephrine MPL: 0.50 REFERENCE: PHARMACOLOGY 4th Ed. Rang pp.16, 158 94. A 25 year old patient weighing 180 lbs consulted at the clinic. She tried several methods of losing weight but failed. Her height is 170 cm. Based on her BMI she is considered: A. healthy C. Obese B. overweight D. underweight MPL: 0.33 REFERENCE: PHARMACOLOGY 4th Ed. Rang p.399 95. This inhibitor of GIT lipase is useful in the treatment of obesity and has minimal side effects: A. sibutramine C. Orlistat B. Phentermine D. glucosan MPL: 0.33 REFERENCE: PHARMACOLOGY 4th Ed. Rang p.407 96. A group of students studied the effects of various GI drugs by measuring the effects on gastric pH and volume. The baseline gastric volume was 2cc and when tested with litmus paper, the paper remained pink. After administering drug A, the gastric volume was 2 cc and the litmus paper turned blue. This means A. The drug is effective in lowering gastric Ph c. B. The drug is effective in increasing gastric pH but no effect on the volume d. C. The drug is effective in lowering the gastric pH and volume e. D. The drug has no effect at all on volume and ph of gastric juice. MPL: 0.50 REFERENCE: PHARMACOLOGY 4th Ed. Rang p.375 97. A patient taking this drug came into the clinic due to visual disturbances, constipation, difficulty in urination and dry mouth. This drug could be:

A. muscarinic antagonist C. dopamine D2 antagonist B. histamine H2 antagonist D. gastrin antagonist MPL: 0.50 REFERENCE: PHARMACOLOGY 4th Ed. Rang p.122 98. The onset and duration of action of NPH insulin are extended because: A. protamine decreases the rate at which insulin is absorbed B. protamine blocks insulin metabolism in the liver C. protamine is basic and combines with insulin by charge interactions f. D. protamine is slowly degraded proteolytically releasing the bound insulin MPL: 0.25 REF. BASIC & CLINICAL PHARMACOLOGY 9th Ed. KATZUNG p.697 99. Which of the following is NOT TRUE about insulin action? A. It stimulates glycogen synthesis in muscle fiber B. It inhibits lipolysis in the adipocyte C. It stimulates K fatty acid synthesis in the hepatocytes g. D. It stimulates gluconeogenesis in the hepatocytes MPL: 0.50 REFERENCE: PHARMACOLOGY 4th Ed. Rang p.387 100. Which of the following drugs is most likely to improve gastric emptying in-patients with diabetic gastroparesis? A. Loperamide C. magnesium hydroxide B. Cisapride D. Sucralfate MPL: 0.50 REFERENCE: PHARMACOLOGY 4th Ed. Rang p.381 End ……………………………………………

Questions

Answer A

1. Extrapyramidal adverse effect is commonly seen with the use of this antipsychotic agent: A. Haloperidol C. Chlorpromazine B. Thioridazine D. Fluoxetine 2. An anti-malarial agent which also exerts an amebicidal effect A. Chloroquine C. Primaquine B. Chloroguanide D. Mefloquine 3. An anti-arrhythmic drug that predominantly block the slow calcium channels A. Verapamil C. Lidocaine B. Amiodarone D. Metoprolol 4. Repeated exposure to which of the following anesthetic agents is most associated with liver damage A. Isoflurane C. Nitrous oxide B. Halothane D. Enflurane 5. The pharmacokinetic parameter which determines the speed of drug input that must balance the speed of drug elimination to achieve a steady state concentration A. clearance C. dosing rate B. bioavailability D. volume of distribution

A

6. If a drug has a high distribution volume, it indicates that: A. it is highly bound to plasma proteins B. drug is highly retained in the vascular compartment C. the plasma drug concentration will increase D. there is extensive tissue uptake

D

7. In what condition are the drugs phenoxybenzamine and phentolamine useful? A. pheochromocytoma C. essential hypertension

A

A

B

C

Reference Goodman and Gilman 10th ed p.488

MPL 1.0

Goodman and Gilman 10th ed p.1084 Goodman and Gilman 10th ed p.949 Goodman and Gilman 10th ed p.351-352 Basic and Clinical Pharmacol ogy 9th Ed. Katzung p.34-41 Basic and Clinical Pharmacol ogy 9th Ed. Katzung p.35 & 47 Basic and Clinical Pharmacol

1.0

0.25

0.33

0.50

0.25

1.0

B. migraine attacks

D. benign prostatic hyperplasia

8. A college friend consulted you regarding the suitability of the therapy his doctor prescribed for severe hypertension. He complained of postural and exercise hypotension (“dizziness”), diarrhea and problems with ejaculation. Which of the following is most likely to produce the effects that your friend has described? A. Propanolol C. Hydralazine B. Guanethidine D. Captopril 9. Norpepinephrine acts as a neurotransmitter at: A. all preganglionic neurons B. parasympathetic postganglionic neurons C. sympathetic postganglionic neurons D. neuromuscular junction

B

10. In the treatment of organophosphate poisoning, the mechanism of action of pralidoxime is: A. competitive inhibition with acetylcholine B. non-competitive antagonism with acetylcholine C. regeneration of acetylcholine D. regeneration of acetylcholinesterase 11. A patient was given digitalis for heart failure, complained of nausea, vomiting, and diarrhea. PE revealed irregular cardiac rhythm. Which of the following is the most appropriate thing to do? A. increase the dose of digitalis B. add a beta-blocker C. discontinue digitalis D. administer potassium 12. A 52 year old man consulted at the clinic due to hypertension (BP 190/90). Except for mild wheezing, his PE findings were unremarkable. The drug given can

D

C

C

C

ogy 9th Ed. Katzung p.144-146 Basic and Clinical Pharmacol ogy 9th Ed. Katzung p.166

0.33

Basic and Clinical Pharmacol ogy 9th Ed. Katzung p.145 Basic and Clinical Pharmacol ogy 9th Ed. Katzung p.103 Basic and Clinical Pharmacol ogy 9th Ed. Katzung p.993

0.33

Basic and Clinical Pharmacol

0.33

0.33

0.25

depress AV conduction, hence the use of beta-blocker is contraindicated. The drug that was given is: A. Methyldopa C. Verapamil B. Enalapril D. Hydrochlorothiazide 13. A patient noted to have elevated cholesterol, LDL, and VLDL. Which among the following drugs act by blocking HMG-CoA reductase to inhibit cholesterol synthesis? A. Probucol C. Lovastatin B. Clofibrate D. Nicotinic acid 14. A 20 year old female was treated for endometriosis for 6 months. She claims to have been given a drug with androgenic effects such as hirsutism, deepening of the voice, and acne. Which of the following drugs could have been given? A. GnRH agonist B. Combined oral contraceptive pills C. Danazol D. Medroxyprogesterone acetate 15. A patient underwent hip replacement and was given morphine for pain relief. At present her RR was noted to be at 8 cycles/min with prolonged episodes of apnea. What will you give this patient to reverse the present condition? A. atropine sulfate C. Naloxone B. Flumazenil D. Protamine sulfate 16. A 7-year old was admitted because of pneumonia. On routine stool exam E. histolytica cyst 5-8/hpf was noted. What will you give? A. Diloxanide furoate C. both A& B B. metronidazole D. no treatment necessary 17. A patient with septic shock was noted to have absent urine output for the past 12 hours. Which of the following drugs would be most helpful to this patient? A. Furosemide C. Dopamine B. Norpeinephrine D. Epinephrine 18. A 40 year old male was seen at the ER because of anaphylaxis immediately given epinephrine. Apparently, the patient has taken prazosin for his

ogy 9th Ed. Katzung p.235-236 C

C

C

Goodman and Gilman 8th ed p.881-884

1.0

Pharmacology 4th ed page 447

0.25

Pharmacology 4th ed Rang Page 598

1.0

A

Pharmacology 4th ed Rang Page 736

0.33

C

Pharmacology 4th ed Rang Page 288

0.33

C

Pharmacology 4th ed Rang

0.33

hypertension. Which of the following may be observed in this patient? A. He may develop hypertensive crisis. B. There will be no effect on his blood pressure C. He may develop hypotension. D. he may develop severe difficulty of breathing due to bronchoconstriction 19. Two drugs A and B have the same mechanism of action. Drug A at a dose of 5 mg produce the same magnitude of effect as drug B at a dose of 500 mg. This means that A. drug B is less efficacious than drug A B. drug A is 100x more potent than drug B C. toxicity of drug A is less than that of drug B D. drug A is more effective than drug B 20. A diabetic patient developed hyperlipedemia (elevated LDL, and triglycerides, normal HDL). Which of the following is least appropriate for the patient? A. Colestipol C. Lovastatin B. Gemfibrozil D. Niacin 21. A patient with tachyarrhythmia and hypertension would be given an antihypertensive. What agent would be the best choice for this patient? A. Felodipine C. Nifedipine B. Isradipine D. Verapamil 22. Which of the following conditions will NOT benefit from the use of vasodilators? A. hypertension C. angina B. migraine D. peripheral vascular disease 23. A patient at the delivery room is having uterine atony after giving birth toan 8-lb baby boy. Her BP at present is 150/90 mmHg. What agent should you give her: A. methyergonovine maleate B. oxytocin C. carboprost D. ergometrine 24. The 2nd generation cephalosporin which crosses the blood brain barrier:

Page 152

B

Pharmacology 4th ed Rang Page 29

0.50

D

Pharmacology 4th ed Rang Page 308

0.25

D

Pharmacology 4th ed Rang Page 275

0.25

B

Pharmacology 4th ed Rang Page 290 Pharmacology 4th ed Rang Page 450

0.50

Pharmacology 4th ed

1.0

B

B

1.0

A. Cefoxime C. Cefoxitin B. Cefuroxime D. Cefoperazone 25. A G2 P1 term, will be given prophylactic antibiotic prior to CS. Which of the following should be recommended? A. Cephalexin C. Cefazolin B. Cefoxitin D. Ceftriaxone 26. After prolonged treatment with penicillin, the patient developed pseudomembranous colitis. What antibiotic will you give for this case? A. Aztreonam C. Imipenem B. Ceftriazone D. Vancomycin 27. This agent inhibits topoisomerase II (DNA gyrase) A. Clindamycin C. Spectinomycin B. Ciprofloxacin D. Tetracycline 28. A neonate was given IV antibiotics for 3 days. However, the baby developed hypothermia, diarrhea and grayish color of the skin. The baby was probably given: A. Amikacin C. Chloramphenicol B. Erythromycin D. Gentamycin 29. The purpose of giving compound drug therapy in the treatment of tuberculosis: A. to rapidly eradicate the strains of tubercle bacilli B. to shorten the infectious phase C. to decrease the emergence of resistant organisms D. to prevent complications of tuberculosis 30. Which of the following is NOT a first line agent in the treatment of tuberculosis? A. Ethambutol C. Rifampicin B. Pyrazinamide D. Streptomycin 32.A patient taking warfarin was also being treated for fungal infection. After 7 days,the patient developed epistaxis. Which of the following antifungal agents was given? A.Nystatin C.Flucytosine B.Ketoconazole D.Amphotericin B 33.If the physician makes a prescription order that utilizes a drug supplied by the pharmaceutical company, what class of prescription order was made? A.Extemporaneous C.Precompounded

C

D

B

C

C

D

B

C

Rang Page 694 Pharmacology 4th ed Rang Page 695 Pharmacology 4th ed Rang Page 702

0.33

0.25

Pharmacology 4th ed Rang Page 720 Pharmacology 4th ed Rang Page 692

0.25

Pharmacology 4th ed Rang Page 706

0.25

Pharmacology 4th ed Rang Page 706 Pharmacology 4th ed Rang Page 317

1.0

Generics Acts of the Philippines

0.25

1.0

0.25

B.Compounded D.Erroneous 34. Which among these antithrombotic drugs inhibit the synthesis of thromboxane by irreversible acetylation of the enzyme cyclooxygenase? A. Abciximab C. Integrilin B. Aspirin D. Ticlopidine 35. A 60 year old was seen at the OPD due to epistaxis which occurred thrice for the past week and easy bruising. He is taking warfarin for AF. Perinent Lab exam: INR-4.5 The INR result is brought about by which of the following? A.increased thrombin activity B.decreased platelet activity C.diminished levels of vitamin K D.decreased levels of factors II, VII,IX and X 36. A 67 year old with atrial fibrillation is currently taking warfarin and digoxin. In the event this patient develops massive bleeding, which of the following would be appropriate for him? A.give vitamin K B.change to LMWH C.give protamine sulfate D.transfuse platelets 37.A 26 year old medical student was brought to the ER for an apparent suicide poisoning due to a broken heart. The maid could not recall the medication but claimed the bottle contained medicine for headache. On PE, the physician noted cyanosis of the oral mucosal, tongue and nail beds. Which of the following is the drug most likely ingested? A.Aspirin C.Flurbiprofen B.Acetaminophen D.Meloxicam 38.A 52 year old woman was suffering from severe joint pains and was diagnosed to have Rheumatoid Arthritis. Apparently, she also appears to be suffering from acid peptic disease. Among the following, which is the safest to give her? A.Aspirin C.Celecoxib B.Ibuprofen D.Phenylbutazone 39.A 21 year old male, took a bottle of insecticide from their garden approximately 30-40 cc in amount. He later developed

B

Pharmacology Goodman and Gilman 10th ed. Pp 1534 Pharmacology 4th ed Rang Page 316

0.50

A

Pharmacology 4th ed Rang Page 319

0.25

B

Pharmacology 4th ed Rang Page 232

0.25

C

Pharmacology 4th ed Rang Page 234

0.50

C

Pharmacology 4th ed

0.25

D

1.0

dizziness, visual blurring, generalized body weakness, numbness, severe epigastric pain and shortness of breath. He was found lying on the floor with stiffening of the trunk and extremities and soaked with sweat. He was rushed to the ER, pertinent findings included drooling of saliva with 3 episodes of non projectile vomiting. Which of the following should be given? A.Pilocarpine C.Atropine D.Physostigmine D.Carbachol 40.This inhibits de novo synthesis of cholesterol by inhibiting HMG CoA reductaseA.Atorvastatin C.Gemfribozil B.Colestipol D.Niacin 41.An asthmatic patient came to the ER because of generalized wheezing in the lungs. Urgent management was done. However, after a while, hypertension was noted. Which of the following medications was probably given? A.salbutamol C.terbutaline B.isoproterenol D.salmeterol 42.A flight attendant has been having allergic rhinitis. She comes to your clinic asking for medication prescription two hours prior to her flight. What will be appropriate for her? A.diphenhydramine C.loratadine B.chlorpheniramine D.promethazine 43.This acts by inhibiting the immediate phase of asthma as it prevents mediator release from mast cellsA.cromolyn sodium C.montelukast B.beclomethasone D.zileuton 44.A patient with severe persistent asthma was treated for six months now with prednisone. Which of the following would be the expected side effect. A.cushingoid facie C.hyperkalemia B.hypoglycemia D.leonine facie 45.FM, 40 year old male was seen at the clinic for check-up. He has bradykinesia, muscle rigidity, pill rolling movement, resting tremors and shuffling gait. The single most effective agent in the treatment of this disorder is: A. Levodopa C. Carbamazepine B. Baclofen D. Tacrine

Rang Page 122

A

B

C

A

A

A

Pharmacology 4th ed Rang Page 305 Pharmacology 4th ed Rang Page 160

0.50

Pharmacology 4th ed Rang Page 345

0.50

Pharmacology 4th ed Rang Page 347 Pharmacology 4th ed Rang Page 423

0.25

Pharmacology 4th ed Rang Page 509

1.0

0.50

0.50

46. Which of the following is a selective inhibitor of MAO-B? A. Selegeline C. Tolcapone B. Amantadine D. Bromocriptine

A

Pharmacology 4th ed Rang Page 511 Pharmacology 4th ed Rang Page 510

0.50

47. Which of the following statement is NOT true regarding Levodopa? A. It is almost always used in combination with decarboxylase inhibitors. B. It has a short half-life about 1 – 3 hrs. C. If administered with meals, its absorption is delayed. D. It has very little side effect and can be withdrawn abruptly. 48. What effect does the use of oral contraceptive (OC) have on a patient with hypothyroidism? A. Low dose OC are unlikely to affect her thyroid function tests. B. Estrogen may falsely increase TSH due to alteration in TSH secretion. C. Estrogen may falsely decrease FT4 index due to alterations in TBG D. Estrogen may falsely elevate total T4 levels due to alterations in TBG 49. If a hypothyroid patient pregnant, how might her levothyroxine replacement be affected? A. She might require an increase in dose. B. She may need a decrease in dose. C. She may need to stop the medication. D. She may have to change levothyroxine to liothyronine. 50. A 62 year old with atrial fibrillation was seen at the ER with light headedness and palpitations associated with exercise usually relieved by rest. At present his heart rate is at 140/min. Which of the following is least likely effective in controlling his heart rate? A. Digoxin C. Verapamil B. Diltiazem D. Atenolol 51. Patients receiving insulin therapy generally require which of the following interventions with progression to end-stage renal disease?

D

C

Pharmacology 4th ed Rang Page 630

0.25

A

Basic and clinical Pharmacol ogy 9th Edition Katzung p 631 Pharmacology 4th ed Rang Page 265

0.25

Basic and clinical Pharmacol

0.25

A

A

0.25

0.25

A. decrease total insulin dose B. increase total insulin dose C. increase frequency of insulin administration D. no change in insulin regimen 52. The inhibitor of GIT lipase is useful in the treatment of obesity and has minimal side effects: A. sibutramine C. orlistat B. phentermine D. glucosan 53. An experiment on GI drugs was done. The baseline gastric volume 2 cc and when tested with litmus paper, The paper remained pink. After administering drug A, the gastric volume was 2 cc and litmus paper turned blue. This means A. drug A is effective in lowering gastric pH B. drug A is effective in increasing gastric pH but no effect on volume C. The drug is effective in lowering the gastric pH and volume D. The drug has no effect on the volume and pH of gastric juice 54.A patient taking this drug came into the clinic due to visual disturbances, constipation, difficulty in urination and dry mouth. This drug could be: A.Muscarinic antagonist B.histamine H2 antagonist C.dopamine D2 antagonist D.gastrin antagonist 55.The onset and duration of action of NPH insulin are extended because: A.protamine decreases the rate at which insulin is absorbed B.protamine blocks insulin metabolism in the liver C.protamine is basic and combines with insulin by charge interactions D.protamine is slowly degraded proteolytically releasing the bound insulin 56. An estrogen receptor antagonist which is useful for postmenopausal breast cancer A. Megastrol C. Flutamide B. Tamoxifen D. Leuprolide

C

B

ogy 9th Edition Katzung p 694 Pharmacology 4th ed Rang Page 407 Pharmacology 4th ed Rang Page 375

0.25

0.25

A

Pharmacology 4th ed Rang Page 122

0.50

B

Basic and Clinical Pharmacol ogy 9th Ed. Katzung p.697

0.25

Basic and Clinical Pharmacol ogy 9th Ed.

0.50

B

57. The agent of choice in the treatment of gestational diabetes A. Sulfonylurea C. Insulin B. Biguanide D. Acarbose 58. A 40 year old bank executive sought consult due to a difficulty of going to sleep but no problem in maintaining sleep. He stopped drinking coffee, eating chocolates, and smoking a month ago. The most appropriate drug for him is: A. Diphenhydramine C. Phenobarbital B. Midazolam D. Clonazepam

C

59. A kidney transplant patient is given a drug for immunosuppression to prevent rejection of the allografted organ. The drug is most likely A. Cyclosprine C. Cyclophosphamide B. Azathioprine D. Vincristine 60. The preferred treatment for a patient who suffers a series without complete recovery is: A. Diazepam C. Succinylcholine B. Chlopromazine D. Ethosuximide

B

61. A company manager is suffering from allergic rhinitis. He has a series of staff meeting so he needs a medication that will not make him drowsy. Which of the following would you give? A. Chlorpheniramine C. Terfenadine B. Hydroxyzine D. Diphenhydramine

C

62. A patient with severe liver disease eventually developed encephalopathy. Which agent is important in the therapy of this condition? A. Lactulose C. Loperamide B. Lactate D. Lorazepam 63. A substance that enhances the probability of ovulation by blocking the inhibitory effect of estrogen and thus stimulating

A

B

D

A

Katzung p.679 Pharmacology 5th ed Rang Page 388 Basic and Clinical Pharmacol ogy 9th Ed. Katzung p.362 Pharmacology 5th ed Rang Page 257

1.0

0.33

0.33

Basic and Clinical Pharmacol ogy 9th Ed. Katzung p.391 Basic and Clinical Pharmacol ogy 9th Ed. Katzung p.266 Pharmacology 5th ed Rang Page 375

0.33

Basic and Clinical

0.50

0.50

1.0

the release of gonadotrophin from the pituitary is: A. Clomiphene citrate C. Tamoxifen B. Danazol D. Progesterone 64. Which of the following combination (diuretic agent and MOA) is CORRECT? A. Furosemide – inhibition of carbonic anhydrase B. Acetazolamide – osmotic effect C. Hydrochlorothiazide – inhibition of Na-Cl symport D. Indapamide – inhibition of Na-K-2Cl symport

C

65. Which of the following combination of diuretic and site of action on the nephron is CORRECT? A. Ethacrynic acid – thick ascending limb of loop of Henle B. Furosemide – proximal tubule C. Hydrochlorothiazide – late distal tubule D. Spironolactone – distal convoluted tubule 66. Which of the following is preferred for a pregnant woman with hyperthyroidism? A. Carbimazole C. RAI B. Methimazole D. PTU

A

67. Drugs A and B act on the same tissue or organ through independent receptors, but with opposite effects. This is known as: A. Competitive antagonism B. Irreversible antagonism C. Physiologic antagonism D. Chemical antagonism 68. This pharmacokinetic value most reliably reflects the amount of drug reaching the target tissue after oral administration A. Volume of distribution B. Area under the blood concentration-time curve C. Peak blood concentration D. Time to peak blood concentration

C

D

B

Pharmacol ogy 9th Ed. Katzung p.681 Basic and Clinical Pharmacol ogy 9th Ed. Katzung p.249 Basic and Clinical Pharmacol ogy 9th Ed. Katzung p.242 Basic and Clinical Pharmacol ogy 9th Ed. Katzung p.632 Goodman and Gilman 8th ed p 53-54

Goodman and gilman 8th edition Pp2627

0.50

0.50

0.33

0.5

0.5

69. A 3 year old patient passed out noodle-like worms and was given an appropriate anti-helminthic. The mechanism of the drug is most probably via A. interference with cell wall synthesis B. inhibition of neuromuscular transmission C. interference with cell wall division D. interference with protein synthesis 70. A patient with nosocomial pneumonia is given ceftazidime and another antibiotic. The most probabale purpose for adding the second antiobiotic is to: A. Increase renal excretion B. Decrease systemic toxicity C. Increase oral absorption D. Prevent the emergence of resistant bacteria 71. About 75% of patients in this condition may suffer from vertigo, inability to perceive termination of movement and difficulty in sitting or standing without visual clues A. Patients allergic to penicillin B. Patients receiving streptomycin therapy C. Patients on isoniazid for tuberculosis D. Patients on amphotericin B 72. A kidney transplant patient is given a drug for immunosuppression to prevent rejection of the allografted organ. The most appropriate drug is: A. Cyclosporine B. Azathioprine C. Cyclophosphamide D. Vincristine 73. A hypertensive patient is treated by a doctor and experiences hemolytic anemia with a positive Coomb’s test. This is most likely caused by: A. Clonidine C. Methyldopa B. Captopril D. Prazosin 74. In a hypertensive patient who is taking insulin to treat diabetes, which of the following drugs should be used with caution? A. Propanolol C. Hydralazine B. Methyldopa D. Prazosin 75. A patient has been taking Imipramine for depression.

B

Goodman and gilman 8th edition pp. 969970

0.5

D

Goodman and gilman 8th edition Pp1127

1.0

B

Goodman and gilman 8th edition Pp1104 1108

0.5

B

Goodman and gilman 8th edition Pp1270

0.5

C

DiPalma 4th ed p. 472

0.50

A

Goodman and Gilman 8th ed p.239

0.5

B

Goodman

1.0

He would most likely experience which of the following adverse effects? A. Seizures C. hepatotxicity B. anticholinergic effects D. nephrotoxicity 76. Chlorpromazine may be used not only in treating schizophrenia but is also effective A. in reducing nausea and vomiting B. as an antihistaminic C. as an antihypertensive agent D. for treating bipolar affective disorder 77. A child suffers from lapses of consciousness or vacant stares lasting about 10 secs, often in clusters. The drug which may be most helpful for his disorder is: A. Phenytoin C. Ethosuximide B. Carbamazepine D. Phenobarbital 78. Upon taking Penicillin, a patient suddenly experiences respiratory distress and circulatory collapse. Which drug would be most helpful in this situation? A. Norepinephrine C. Isoproterenol B. Epinephrine D. Atropine 79. A gout patient developed nephropathy characterized by overproduction of uric acid and extreme hyperuricemia. What agent could have helped prevent this? A. Cyclophosphamide C. Sodium chloride B. Allopurinol D. Antidiuretic hormone 80. A patient with peptic ulcer and on therapy with cimetidine may experience decreased effects of many other drugs because cimetidine is an inhibitor of A. the proton pump C. monoamine oxidase B. tyrosine kinase D. cytochrome P-450 73. The index of safety of a drug decreases as the median toxic dose: A. decreases and the median effective dose increases B. increases and the median effective dose decreases C. the median effective dose decreases D. the median effective dose increases 74. What is the major second messenger of beta receptor

and Gilman 8th ed p.405-414 A

activation that participates in signal transduction? A. inositol triphosphates C. calcium B. cAMP D. adenylyl cyclase

DiPalma 4th ed p.275-277

0.5

C

Goodman and Gilman 8th p. 449 – 453

0.5

B

Goodman and Gilman 8th ed p.198

1.0

B

Goodman and Gilman 8th ed p.678

1.0

D

Goodman and Gilman 8th ed p.901

1.0

Basic and Clinical Pharmacol ogy 9th Ed. Katzung p.30 Basic and

0.33

A

B

0.5

75. Which of the following bronchodilators is a xanthine derivative? A. Terbutaline C. theophylline B. ipratropium bromide D. salmeterol

C

76. Myelosuppression is a common side effect associated with the use of the following anticancer drugs, EXCEPT: A. Vincristine C. Methotrexate B. Cyclophosphamide D. Chlorambucil

A

77. The following anticancer drug, with its toxic effect, is correctly matched with the appropriate antidote: A. cyclophosphamide-induced hemorrhagic cystitis – ondansetron B. methotrexate-induced myelosuppression – MESNA C. cisplatin-induced nausea and vomiting – leucovorin D. doxorubicin-induced arrhythmia - dexrazoxane 78. Which of the following anticancer drugs act as a mitotic inhibitor? A. Methotrexate C. Bleomycin B. Vinblastine D. Carmustine

D

79. Radical cure for malaria is achieved by giving: A. Chloroquine C. Mefloquine B. Quinine D. Primaquine

D

B

Clinical Pharmacol ogy 9th Ed. Katzung p.25-27 Basic and Clinical Pharmacol ogy 9th Ed. Katzung p.324, 325, 328 Basic and Clinical Pharmacol ogy 9th Ed. Katzung p.911 Basic and Clinical Pharmacol ogy 9th Ed. Katzung p.914 Basic and Clinical Pharmacol ogy 9th Ed. Katzung p.911 Basic and Clinical Pharmacol ogy 9th Ed.

1.0

0.33

0.50

0.75

1.0

80. A 3 year old was brought to the ER because of recurrent attacks of blank stares lasting for a few minutes. The anti-seizure agent that you will give the child is: A. Phenobarbital C. Lamotrigine B. Ethosuximide D. Vigabantrine 81. The mainstay drug in the treatment of all types of leprosy is: A. Clofazime C. Isoniazid B. Dapsone D. Rifampicin 82. A patient on anti-TB drug developed peripheral neuritis. This was improved by giving Pyridoxine. The drug taken must have been: A. Ethambutol C. Isoniazid B. Rifampicin D. Para-aminosalicylic acid

B

73. A patient with elevated cholesterol level is also taking warfarin for his deep venous thrombosis. The lipid lowering agent that may enhance the effect of Warfarin is: A. nicotinic acid C. cholestyramine B. lovastatin D. gemfibrozil

C

74. A known diabetic patient was brought to the ER unconscious with random blood sugar of 900 mg/dl. ABG study showed metabolic acidosis. Which of the following preparation is appropriate? A. regular insulin B. insulin zinc preparation C. isophane zinc suspension D. potassium zinc insulin suspension 75. Which of the following corticosteroids has the greatest anti-inflammatory potency relative to cortisol? A. Prednisone C. Aldosterone B. Dexamehtasone D. Triamcinolone

A

B

C

B

Katzung p.871 Goodman and Gilman 10th ed p.535

0.25

Goodman and Gilman 10th ed p.1288 Basic and Clinical Pharmacol ogy 9th Ed. Katzung p.784 Basic and Clinical Pharmacol ogy 9th Ed. Katzung p.574 Basic and Clinical Pharmacol ogy 9th Ed. Katzung p.700

1.0

Basic and Clinical Pharmacol ogy 9th Ed.

0.5

1.0

0.5

0.75

76. A 30-year old G2P1 patient suffers experiences a regular and increasingly frequent contractions at 6 months of gestation. Which of the following agents may help in this case? A. Terbutaline C. Propanolol B. Nicotine D. Tacrine

A

77. The primary mechanism underlying the resistance of gram positive organism to macrolide antibiotic is: A. methylation of binding site on the 50S ribosomal subunits B. formation of esterases that hydrolyzed the lactone ring C. formation of drug-inactivating acetyltransferase D. decrease drug permeability of the cytoplasmic membrane 78. A prescription that has the generic name enclosed in a parenthesis and written below the brand name can be interpreted as: A. violative prescription B. imposiible prescription C. erroneous prescription D. correct prescription 79. The onset and duration of action NPH insulin are extended by protamine because: A. it decreases the rate of insulin absorption B. it blocks insulin metabolism in the liver C. it is a basic compound and combines with insulin by charge interactions D. it is slowly degraded proteolytically releasing the bound insulin 80. The following anti-anginal drugs are vasodilators, EXCEPT: A. Felodipine C. Metoprolol B. Nitroglycerin D. Diltiazem

A

Katzung p.647 Basic and Clinical Pharmacol ogy 9th Ed. Katzung p.138 Goodman and Gilman 10th ed p.1250

1.0

0.25

C

Generic Act of the Philippines

0.5

B

Pharmacology 4th ed Rang Page 697

0.25

C

Basic and Clinical Pharmacol ogy 9th Ed. Katzung p.186

0.50

81. Which is a direct effect of nitrates when given in the usual doses for the management of angina? A. increased preload B. increased afterload C. increased cardiac contractility D. decreased preload

D

73. The mechanism of action of Omeprazole A. inhibits H+-K+ ATPase pump B. promotes prostaglandin formation C. blocks action of histamine of H2 receptors D. neutralizes acid by increasing the pH of gastric secretion

A

Basic and Clinical Pharmacol ogy 9th Ed. Katzung p.190 Goodman and Gilman 8th ed p.902-904

1. 3rd generation cephalosporin that does not cross the BBB a. Cefoperazone (and Cefipime) b. Ceftazidime c. Moxolocam d. cefetamet e. all of the above 2. associated with pseudomembronous colitis; a. methicillin b. Pen G Na c. Oxacillin d. Ampicillin e. Ticarcillin 3. Tetracyclines are contraindicated in children because it may causes; a. GI irritation b. Hepatotoxicity c. Bone and tooth discoloration d. Renal toxicity e. Only A and B 4. Anticonvulsants that have been shown to have antimanic effects; a. Carbamazepine b. Valproic acid c. Phenytoin

0.50

1.0

d. All of the above e. A and B only 5. The major working hypothesis for schizophrenia, and the basis for much of drug therapy is the; a. Serotonin hypothesis b. Omine hypothesis c. Dopamine hypothesis d. Muscarinic hypothesis e. Serotonim hypothesis 6. Causes harmless orange color of urine ,sweat and saliva; a. INH b. Rifampin c. Ethambutol d. Cycloserine e. Ethionamide 7. This anticancer causes a minimal bone suppressant effect; a. Vincristine b. Methotrexate c. Cyclophosphamide d. Cytarabine e. Doxorubucin 8. This is the drug of choice for mixed round worm infection; a. Mebendazole b. Pyrantel pamoate c. Niclosomide d. Thiobendazole e. Metronidazole 9. The fraction of the drug that reaches the general circulation is; a. Bioavalability b. Bioequivalence c. Biodisposition d. Biotransformation e. First pass 10. This deals with the biochemical and physiological effects of drugs and their mechanism of action a. Pharmacokinetics b. Pharmacodynamics

c. Pharmacotherapeutics d. Pharmacogenetics e. Pharmacy 11. Which one of the following pairs of drugs has been known to cause a lupus like syndrome a. Guarenthidine and reserpine b. Lidocaine and phenytoin c. Procainamide and hydralazine d. Proprendol and disopyramide 12. Which one of the following statements is false about furosemide? It a. Increases the excretion of Na, Cl, and water b. Increases the excretion of K c. Increases the excretion of uric acid d. May have to be used in conjunction with K sparing diuretic 13. The treatment of ketoacidosis may include the administration of a. CaCl b. Crystalline zinc insulin c. Glucagons d. ZnCl 14. The action of cortisone is characterized by a. Causing hyperglycemia b. Decreasing gluconeogenesis c. Enchancing peripheral utilization of glucose d. Having an antiglucagon effect 15. Methimazole exerts its effects by inhibiting; a. Iodide acidification b. Iodide transport and accumulation c. The formation of diiodothyrosine d. The tubular secretion of iodine 16.

16.

Pharmacological properties of 5HT except:

a. Enhances GIT motility b. Increased force of contraction in the heart c. Bronchoconstriction d. Bronchodilation Piperazine derivatives except a. Hydroxyzine b. Cetirizine

c. Meclizine d. Carbocisteine (an opioid) 17. They do not undergo the process of phosphorylation to exert their anti viral activity a. Indinavir (protease inhibitor) b. Cidofavir c. Epivir d. Lamivudine 18. The following drugs reduce platelet aggregation by inhibiting the ADP pathway a. Clopidogrel (& Ticlopidine) b. Abciximab c. Aspirin 19. Which drug decreases plasma triglycerides by increasing lipoprotein lipase; a. Lovastatin b. Nicotinic acid c. Cholestyramine d. Probocol e. clofibrate 20. Inhibits phosphodiesterase enzyme a. Salbutamol b. Aminophylline c. Cromolyn sodium d. Ephedrine e. B & C 21. Agent of choice in congestive heart failure which are superior to other vasodilators a. Felodipine b. Spironolactone c. Enalapril d. Digoxin 22. Occurs during phase 2 of action potential of Purkinje fibers a. Upstroke ends as Na channels are rapidly inactivated b. K –channels rapidly open and close causing a transient outward current

c. Voltage-sensitive Ca-channels upon resulting in slow inward current d. K-channels open resulting outward current 23. Orally given in patients with severe left ventricular systolic dysfunction a. Digoxin b. Dobutamine c. Oraninone d. A and B only 24. An agent that neutralizes HCI secreted by parietal cells and increases gastric motility a. Aluminum hydroxide b. Magnesium hydroxide c. Calcium carbonate d. Sodium bicarbonate e. Calcium chloride 25. Contraindications for aspirin use except; a. Elderly b. PUD c. Pregnancy d. Hemophilia e. Liver damage 26. A drug used in the treatment of gout that reduces uric acid synthesis by blockage of hypoxanthine; a. Indomethacin b. Aspirin c. Colchicines d. Allopurinol e. Sulfinpyrazone 27. The following is are True about Naftifine; a. Interferes with ergosterol biosynthesis b. Inhibits squalene epoxidase c. Effects CYP 450 enzyme system d. Significant drug interact e. A and B only 28. Inhibit DNA polymerase enzymes a. Didanosine

b. Stavudine c. vidarabine d. AZT e. A and C only 29. Specific 5HT3 antagonist used for nausea and vomiting secondary to chemo therapy a. Methylgerside b. Ondansetron c. Metoclopromide 30. H1-receptor antagonist is which has anti-cholinergic properties except a. Diphenhydramine b. Promethazine c. Meclizine d. Estemizole 31. A patient was diagnosed as suffering from chronic refractory congestion and oliguria. Which one of the following agents is most effective? a. Acetylcholine b. Atropine c. Dopamine d. Terbutaline e. Propanolol 32. The nonselective beta-adrenergic blocking agent that is also a competitive alpha-adrenoceptors a. Timolol b. Madolol c. Pindolol d. Labetolol e. Esmolol 33. All of the following compounds are direct-acting cholinergic agonist, except a. Acetylcholine b. Pilocorpine c. Edrophonium d. Bethanecol 34. Atropine and its analogs produce which of the following effects;

a. Pupillary constriction b. Increased gastric acid secretion c. Decreased secretion in the respiratory tract d. Increased peristalsis e. Bronchoconstriction The most potent of the inhalational anesthetics is; a. Enflurane b. Halothane c. Isoflurane d. Methoxyflurane An ester local anesthetic with vasoconstrictive properties: a. Lidocaine b. Tetrocaine c. Bupivacaine d. Cocaine e. Procaine Drug of choice for management of acid hypersecretion associated

35.

36.

37. with 38.

39.

40.

41.

a. Cholinergic agonist b. Prostaglandin analog An antiarrhythmic drug that acts on K-channels a. Esmolol b. Sotolol c. Diltiazem d. Adenosine Prolongs phase 3 repolorization a. Na-channel blocker b. B-adrenareceptor blocker c. K-channel blocker d. Ca-channel blocker True statements about lidocaine except a. Use for ventricular arrhythmias arising from MI b. Does not markedly slow conduction c. More effective on atrial or AV junction arrhythmias d. None of the above Calcium channel blocker which can relieve coronary spasm

a. Mifedipine b. Verapomil c. Diltiazem d. Hydralazine 42. Commonly used antihypertensive drugs in patients with insulin dependent diabetes mellitus a. Calcium channel blockers b. ACE inhibitors c. Both of the above 43. Effects of angiotensin II on adrenal cortex a. Stimulates catecholamine biosynthesis b. Stimulates mineralocorticoid synthesis c. Renal vasoconstriction 44. Specific for angiotensin AT receptors a. Captopril b. Candesertan c. Voscose d. A and B only 45. Overuse of loop diuretics will cause which of the following a. Hypokalemic metabolic alkalosis b. Hyperkalemic metabolic acidosis c. Hypokalemic metabolic acidosis d. Hyperkalemic metabolic alkalosis 46. The above condition is due to the secretion of a. Sodium and hydrogen ions b. Potassium and hydrogen ions c. Sodium and chloride ions d. Potassium and chloride ions 47. Which of the following should not be given to a hypertensive a. Lisinopril b. verapamil c. methyldopa d. Hydrochlorothiazide 48.Inhibits platelet aggregation by inhibiting ADP pathway a. Aspirin b. Heparin c. Warfarin d. Ticlopidine

49.

Inhibits ligand binding to platelet GP IIb/IIIa receptor by occupancy a. Abxicimab b. Tirofiban c. Enoxaparin d. Anistreplase 50. Analog of aminocaprioc acid that competitively inhibits plasminogen activated a. Tranexamic acid b. Eprotinim c. Reteplase d. Warfarin 51. Effective as a singe drug therapy for familial dysbetalipoproteinemia a. Fibric acid b. Bile acid-binding resin c. HMG-CoA reductase d. Probucol 52. Most common side effect of antihyperlipidemic agents a. Elevated blood pressure b. Neurological problems c. Gastrointestinal disturbance d. Migraine headaches 53. Causes decrease in liver triacylglycerol synthesis by limiting available free fatty acid needed as building blocks for this pathway a. Cholestyramine b. Elefibrate c. Levostatin d. Niacin 54. The following drugs maybe used during acute exacerbations of bronchial asthma except a. Terbutaline b. Salmeterol c. Albuterol d. Epinephrine 55. Effective as prophylactic anti-inflamatory agent a. Nedocromil b. Ipratropium c. Theophylline

d. Loratidine 56. Most reabsorption occurs at what segment of the nephron a. Distal convoluted tubule b. Loop of Henle c. Collecting duct d. Proximal convoluted tubule 57. Are sulfonamide derived diuretic which inhibits Na+/CI- cotransporter at the DCT a. Ethocrymic acid b. Acetazolormide c. Chlorthiazide d. Triamtereme 58. Gynecomastia is an adverse effect of this diuretic a. Mannitol b. Spironolactone c. Chlorthalidone d. Bumetinide 59. H2 receptor antagonist eliminated principally by the kidney hence has 100% bioavailability a. Cimetidine b. Nizatidine c. Famotidine d. Ranitidine 60. A target plasma theophylline concentration of 5mg/l is desired to relieve acute exacerbation of bronchial asthma in a 70 kg patient. If the mean clearance of this drug is 2800 ml/kg, what is the dose rate to be given via IV? a. 7 mg/h b. 12mg/h c. 28 mg/h d. 14mg/h (2.8L/kg X 70kg) 61. Exacerbation was relieved and you want to maintain the plasma level rising oral theophyline every 12 hours using an extended release formula. If oral availlability is 96% what is the maintenance dose? a. 175 mg/dose b. 350 mg/dose c. 700 mg/dose

d. 700 mg/day 62. Single most important factor determining drug concentration a. Clearance b. Half-life c. Absorption d. Volume of distribution 63. A 15 year old maintained on phenobarbital was diagnosed to have typhoid fever. Which of the following antibiotic dose should be increased to achieve a therapeutic dose? a. Trimethoprim b. Sulfamethoxazole c. Chloramphenicol d. Ofloxacin e. Paracetamol 64. MG. 38 year old female, 180 lbs, wanted to reduce weight by taking grapefruit juice. Which of the following drug metabolism will be inhibited? a. Astorvastatin b. Cisapride c. Dicumerol d. A and B only e. All of the above 65. Active component of the above substance inactivates a. Intestinal CYP3A4 b. Hepatic CYP3A4 c. Both d. Neither 66. This/these refer/s to the concentration or those of drug required to produce 50 maximal effect a. Potency b. Efficacy c. Graded dose response d. Quintal dose effect 67.Response to a drug diminishes rapidly after administration of a drug a. Tolerance b. Tachyphylaxis c. Idiosyncracy

d. Median effect A synthetic derivative of glycyrrhizic acid that shows effective healing of gastric duodenal ulcers: a. Sucrolfate b. Carbenaxolone c. Octreotide d. Pirenzipine 69. Emulsifies with stool making passage easier a. Docusate b. Lactulose c. Bisacodyl d. Phenolphthalein 70. Zileuton inhibits lipoxygenase pathway by a. Inhibiting 5-lipoxygenase b. Antagonizing LTD4-receptor c. Alpha adrenoreceptor blockade d. Inhibiting phospholipase A2 71. Glucose transporter seen in muscle and adipose tissue which is responsible for intake uptake of glucose. a. GLUT 4 b. GLUT 3 c. GLUT 2 d. GLUT 1 72. Long acting insulin/s a. Lente humulin b. Insulin lispro c. Ultralente humulin U 73. Beta-lactam antibiotic of choice for enterobacter infections: a. Tazobactom b. Meropenem c. Aztreonom d. Vancomycim 74.Tetracyclines maybe used to the following organisms except a. Mycoplasma pneumoniae b. Entameoeba histolytica c. Neisseria gonorrhea d. Helicobacter pylori 68.

75. Inhibits translocation by irreversibly binding to 50S ribosomal subunit a. Azithromycin b. Gentamycin c. Fluroquinolone 76.A tissue Schizonticide a. Chloroquine b. Quinine c. Mefloquine d. Primaquine 77.Effective single agent for suppressing and curing multidrug resistant P. falciparum a. Quinine b. Mefloquine (???NOTA!) c. Pyrimethamine d. Primaquine 78.Spectrum of activity of ribavirin a. Rhinoviruses b. Enteroviruses c. Influenza viruses d. Hepatitis 79.Dideoxynucleoside that terminates the synthesis of proviral DNA chain and inhibits the RNA transcriptase of HIV and Hepatitis B virus a. Stavudine b. Lamivudine c. Zalcitabine d. Saquinavir 80.Alkylating agents used as anticancer drugs except a. Paclitaxel (Mitosis inhibitor) b. Cyclophosphamide c. Nitrosureas d. Mechlorethamine 81.A co-transmitter at inhibitory enteric nervous system neuromuscular junction that is important at sphincters: a. GABA b. Golanin c. 5 HT d. NO

e. CGRP 82. An autonomic receptor which results to opening of potassium channels and inhibits adenyl cyclase when activated a. MI b. M2 c. M3 d. NN e. NM 83. An alpha adrenergic receptor blocker a. Phentolamine b. Isoproterenol c. Norepinephrine d. Betanechol 84. A direct acting cholinomimetic drug which is/are alkaloid or synthetic analog a. Carbonic acid b. Pilocarpine c. Carbachol d. All of the above 85. The drugs above causes a. Cillary muscle relaxation b. Negative chronotrophy c. Positive chronotrophy d. Vasoconstriction at low dose 86. After a vehicular accident , the patient developed neurogenic bladder. A cholinomimetic commonly used in this case is a. Betanechol b. Nicotine c. Muscarine d. Neostigmine 87. Before giving the drug above you must make sure that a. There is no mechanical obstruction b. The level of cord injury is exact c. The patient had urinary retention d. B and C e. All of the above

88. A new direct acting muscarinic agonist used in the treatment of dry mouth associated with Sjogren syndrome a. Embenorium b. Denepezil c. Cevimeline (?di ko rin mabasa) d. Tacrine e. Pyridostigmine 89. A semisynthetic derivative of plant alkaloid podophyllatoxin blocking cells in the late S-G1 of cell cycle a. Etoposide b. Procarbazine c. L-asparaginase d. Cisplatin 90. True of aspirin a. Reversible acetylators of cyclooxgenase b. Depresses pain stimull at subcortical sites c. Antinflammatory, antipyretic, antiplatelet, analgesic d. Responsible for keeping PDA open 91. An imidazopyridine derivative with hypnotic effects and facilitates GABA mediated inhibition a. Zolpidem b. Buspirone c. Zaleplon d. Flumazeoil 92. Which of the ff. drugs is generally considered the drug of choice in treating status epilepticus a. Phenoborbital b. Amoborbital (Amstal) c. Phenytoin (dilantin) d. Paraldehyde e. Diazepam (Valium) 93. The antihypertensive effect of guanethidine (Ismelin) is inhibited by a. Diazepam (Valium) b. Amitriptyline c. Hydrochlorothiazide (Hydro DIURIL) d. Probenecid (Benemid) e. Nitrofurantoin (furodentin)

94. A physician has decided upon a course of tetracycline therapy for a patient with renal impairment. Which of the following drugs is LEAST likely to accumulate in the blood a. Demeclocycline b. Doxycycline c. Minocycline d. Oxytetracycline 95.Benztropine (Cogentin) is often given to patients taking the antipsychotic phenothiazine. Benztropine: a. Reduces the doze of phenothiazine required b. Is an anticholinergic drug that reduces the extrapyramidal side effects of phenothiazine c. Eliminates the unpleasant GI irritation caused by the phenothiazine d. Is an antidepressant e.reduces gut motility to ensure that the phenothiazine is completely absorb 92. The metabolism of which of the following compounds is altered in patients taking antimetabolite

a. b. c. d. e.

Pyridoxine Folic acid Riboflavin Rennin Tyroxine

93. Which of the following antifungal agents is ineffective against candida organism? a. nystatin (mycostatin) d. halopragin (halotex) b. clotrimazole (lotrimin) e. miconazole c. tolnaftate (tinactin) 94. The clinical investigation of a new drug consists of four phases. Phase I of the clinical testing involves administering the drug. a. to animals to determine side effects of the drug b. to animals for toxicity studies c. by select clinicians to healthy volunteers d. by select clinicians to patients suffering from the disease e. by general practitioners to patients suffering from the disease 95. Tubocurarine should NOT be used in patients who are taking a. aspirin

b. morphine c. indomethacin d. levodopa e. gentamicin 96. Penicilloyl-polytysine is a substance used to: a. stabilized crystalline penicililin G preparations b. counteract allergic reaction to penicillin c. reduce the renal secretion of penicillin d. skin test patients for penicillin allergy e. manufacture the semisynthethic penicillins 97. Ticarcillin may be preferred to carbenenicillin for patients with congestive heart failure, renal failure, and hypertension because it a. contains less sodium than does corbenicillin b. is usually given in smaller doses than carbenicillin c. does not induce hypertension d. stimulates renal blood flow e. has a broader spectrum than does carbenicillin 98. A Fanconi-like syndrome has been associated with the use of outdated and degraded a. ampicillin b. cloxycycline c. tetracycline d. chloromphenicol e. clindemycline 99. Which of the following is true of lithium carbonate (Eskolith, Lithane) a. Indicated in the treatment of severe manic-depressive psychoses b. May only be administered by the intremuscular route c. Onset of action occurs within 2 hours of the first administered dose d. Should be administered with a diuretic to minimize edema formation 100. Patients on lithium carbonate therapy should be advised a. to limit water intake b. to stop taking the drug if they experience mild side effects c. not to restrict their normal dietary salt intake d. not to take the drug during the manic phase of their cycle e. not to take the drug with food 101. Which of the following drugs can interfere with the diagnosis of pernicious anemia a. Pyridoxine b. Menadione c. Thiamine d. Ascorbic Acid e. Folic Acid 102. Which of the following agents would be most dangerous to use in a patient with high doses of gentamicin?

a. ethacrymic acid (Edecrin) b. tetracycline HCI c. propantheline bromide (pro-banthine) d. hydroOIURIL e. pentobarbital sodium 103. Diethylstilbestrol – induced vaginal adenocarcinoma is an example of a. Delayed irreversible toxicity b. dose – dependent toxicity c. hypersensitivity reaction d. idiosyneratic reaction 104. Ethylene glycol poisoning would lastly manifest as a. acidosis b. alkalosis c. oxalate crystals in the urine d.renal insufficiency 105. Isopropyl alcohol poisoning resembles the poisoning produced by which one of a. ethyl alcohol b. ethylene glycol c. kerosene d. methyl alcohol 102. Benzene toxicity is characterized by a. anemia b. otoxia c. gastrointestinal symptoms d. respiratory alkolocis 103. Immunnosuppressive agents such as prednisone and azothioprine have been..for all of the following disorders, except which one? a. active hepatitis b. megalobiastic anemia c. thrombocytopenic purpura 104. The metabolite of azathioprine is a. Cloflucarbon b. 6-mercaptopurine c. merodiceire

d. triclosamide 105. The chief side effect of azathioprine is

d. sulfoxone 111. Primaquine is characterized by the following characteristic, except which one ?

a. alopecia

a. attacks plasmodia in exoerythrocytic stages

b. leucopenia

b. is effective for malarial prophylaxis

c. hypertrichosis

c. may cause hemolytic anemia in susceptible patients

106. The toxicity of trimethadione is manifested as which of the following a. alopecia

d. may cause polycythemia vera 112. Thiabendozole is characterized by the which of the following

b. bone narrow depression

a. exerts its effects by inhibiting fumarate reductase

c. hemerolopia

b. is metabolized by dehydration

107. One reason nitrosoureas are effective in the treatment of brain tumors

113. Third generation cephalosporins such as cefotaxime and ceftazidime

a. are lipid soluble

a. are effective in treating infection caused by aerobic gram-negative

b. chelate carbonium ions

b. do not require dose adjustment in patients suffering from renal imparement

c. chelate isocyanate molecules

c. have identical pharmacokinetic properties

d. inhibit the generation of carbonium ions

d. require dose adjustment in patients with hepatic diseases

108. 5-Fluorouracil is given intravenously because it is a. a gastric secretogogue b. metabolized rapidly in the liver c. not absorbed orally d. a potent irritant to the gastric mucous 109. In brain tumors, which one of the following cells is deficient? a. astrocytes b. eosinophils c. monocytes d. T4- helper lymphocytes 110. Leprosy is treated mainly with a. guenobenz b. Guanfocine c. methylphenidate

114. All of the ff. chemotherapeutic agents have been properly matched with their characteristics except one, which one? a. azlocillin – is less potent than carbenicillin b. carbenicillin indanyl- is effective orally c. mezlocillin – is effective in Klebsiella infection d. ticarcillin – is more potent than carbenicillin 115. Which one of the following penicillin derivatives is not stable in gastric acid a. amoxicillin b. dicloxicillin c. penicillin G 116. Chloramphenicol is characterized by all of the following actions, except which one? a. bacterial resistance results from the development of chloramphenicol acetyltransferases b. it has a narrow spectrum of activity c . it is a drug of choise in the treatment of salmonella infection

d. it is used for treating meningococoal onfection in patients allergic to penicillin 118. A 30 year old overweight woman suffered from paroxysmal supraventricular tachycardia and also borderline hypertension (145/90mml-ig). For which she was not taking any medications. The cardiologist should treat the tachycardia with which one of the following a. digitals followed by quinidine b. propanolol c. propanolol plus thiazide d. verapamil 118. All of the following agents participate in secretion in the exocrine pancreas, except a. acetylcholine b. cholecystokinin c. oxytocin d. substance P 119. All of the following agents belong to the phenylalkylamine – derived calcium entry family except one, which one a. gallopomil b. oxodipine

122. The pertussis toxin a. alters the activity of adenylate cyclase b. resembles botulinum toxin in action c. resembles the cholera toxin in action d. resembles forskolin in action 123. Activation of all of the ff. receptor sites causes stimulation of adenylate cyclase except one, which one? a. alpha2- adrenergic receptors b. beta- adrenenergic receptors c. D1 dopaminergic receptors d. serotonerg ic receptors 124. Which one of the ff. best describes the actions of morphine and enkephalin on adenylate cyclase a. activate adenylate cyclase in therapeutic doses b. inhibit adenylate cyclase in toxic doses c. produce a selective and naloxone-reversible inhibition of adenylate cyclase d. produce a nonspecific and naloxone-insensitive activation of adenylate cyclase 125. Which one of the ff. substances does not increase the concentration of cyclic AMP

c. ronipamil

a. aminophylline

e. verapamil

b. epinephrine

120. All of the ff. agents are dihydropyridine derived calcium entry blockers: a. anipamil b. nicardipine

c. glucagon d. insulin 126. Renal cell carcinoma-induced hypercalcemia may be reduced by the administration of the ff. agents?

c. nifedipine

a. aspirin

d. riodine

b. calcitonin

121. Which one of the ff. is the least recommended indication for verapamil a. atrial flutter b. atrial fibrillation

c. parathyroid hormone d. sodium citrate 126. The prostacyclin-induced secretion of renin is shared by which one of the ff.

c. digitalis toxicity

a. acetylsalicylic acid

d. reentrant paroxysmat supreventricular

b. indomethacin

c. meclofenomate e. norepinephrine 127. The bronchodilating action of prostaglandin E1 is shared by a. acetylcholine b. bradykinin c. histamine d. protoglandin E2 128. In a patient recovering from an acute myocardial infection who is taking warfarin the administration of clofibrate may

131. Following the administration of nitroglycerin, the change in cardiac hemodymanic is due to a. constriction of collateral vessels b. decreasing of myocardial oxygen consumption c. shifting of the blood away from ischemic areas d. venous dilation then increased venous return then rapid ventricular volume with increased wall tension; 132. Antitussive preparations should be used a. in productive cough b. in unproductive cough

a. decrease the risk of hemorrhage

c. only when hypoxia is imminent

b. have no effect at all

d. in copious secrotory cough

c. increase the risk of hemorrhage d. lower the vitamin K level 128. Alpha-tocopherol

133. All of the following drugs have antitussive properties except which one a. codeine b. deserpidine

a. causes thromboembolic diseases

c. dextromethorphan

b. inhibits platelet aggregation

d. morphine

c. is a natural oxidant (hindi daw natural) d. is similar to alpha-interferon 129. Streptokinase and urokinase are contraindicated in the presence of

134. Which one of the following phenothiazine derivatives has no antiemetic effect? a. chlorpromazine b. prochlorperazine

a. arteriovenous shunt

c. thioridazine

b. malignancy

d. triflupromazine

c. pulmonary embolism d. thrombosis 130. Transderm- Nitro is contraindicated in:

135. Which one of the ff. gastrointestinal agents does not stimulate parietal cell function a. acetylcholine b. gastrin

a. effort angina

c. histamine

b. elevated ocular and cerebrospinal fluid pressure

d. prostaglandin

c. stable angina d. variant angina

136. Which one of the following compounds has both estrogenic and antrestrogenic properties a. chlorpromazine

b. clofibrate

b. doxorubicin

c. clomiphene

c. dactinomycin

d. clonidine

d. cyclosporine

137. Which one of the ff/ agents inhibits spermatogenesis?

142. Urine monoclonal antibody to CD3 antigen on the surface of human thymocytes

a. gelusil

a. levarmisole

b. gemcodial

b. hymphocyte immune globulin

c. gestodene

d. muromonab

d. gossypol 138. The cellular metabolism of calcium is characterized by

143. Elective MAO type B inhibitor used as adjunct to parkinsonism a. amantadine

a. calcium pumps maintaining the differential intracellular versus extracellular calcium

b. selegiline

b. being unaffected by anticonvulsants

c. bromocriptine

c. the intracellular calcium being bound only to organic phosphate

d. haloperidol

d. the intracellular concentration of calcium being 1,000-fold higher than the extracellular concentration of calcium 139. Strategy in cancer chemotherapy wherin cancer cells are held at M phase followed by administration with another CCS drugs a. pulse therapy b. recruitment c. synchrony d. rescue therapy 140. Cell cycle specific drug that acts in late S and Go phases which are used as a component of combination therapy for testicular carcinoma a. etoposide b. flutamide c. leuprolide d. mercoptopurine 141. Peptide antibiotic immunosuppressive which inhibits early stages of differentiation of T cells and blocks their activation a. bleomycin

144. Patient was diagnosed to have Huntington disease, his physician wishes to give drug, the best to give is/are. a. tetrobenazine b. haloperidol c. phenothiazine d. all of the above 145. A pro-drug that acts on platelets: a. heparin b. enoxaparin c. warfarin d. ticlopidine 145. Simvastatinand prevastatin will a. increase clearance of IDL and LDL b. reduce VLDL secretion from the liver c. decrease VLDL by stimulation of lipoprotein lipase d. reduce LDL by an unknown mechanism

146. A weak acid that compete with uric acid in renal tubule for reabsorption

b. labetalol

a. allopurinol

c. phentolamine

b. sulfinpyrazone

d. isoproterenol

c. phenocitine d. acetazolamide 147. An antithyroid drug that inhibits organification and thyroid hormone release

148. Which of the ff. drugs of choice in anaphylaxis associated with bronchospasm and hypotension? a. cortisone b. epinephrine

a. methimazole

c. isoproterenol

b. Lugol’s solution

d. phemytephrine

c. I131

e. terbutolone

d. propanolol 148. A partial agonist that binds progestin androgen and glucorticoid receptors used in a. clomiphene b. danazol c. mifepristone d. relaxin 149. True of proinsulin a. an 86- amino acid single chain polypeptide b. cleavage and cross linking result in one chain 51 peptide insulin molecule c. can be measured by immunoassay independently of insulin d. all of the above 146. Streptomycinc an be used against all except a. tuberculosis b. tularemia c. serratia d. brucella 147. A 88 year old man came in because of difficulty in starting to urinate PE revealed a BP 160/100 mmHg & a slightly enlarged prostate. Which of the ff. medications is useful in treating both conditions? a. alfuzosin

149. A 40-yr old patient presents with an acute hypertensive crisis. Which agent will act to block the increased blood pressure by acting at sympathetic ganglia? a. propranolol b. trimetaphan c. nitroglycerin d. digoxin e.labetalol 150. A patient was diagnosed as suffering from chronic refractory congestive heart failure with oliguria. Which one of the ff. agents is most effective? a. acetylcholine b. atropine c. dopamine d. terbutaline e. propranolol 151. A patient with history of bronchial asthma can be given a selective B1 adrenergic receptor blocker a. propronolol b. nadolol c. timolol d. metoprolol e. pindolol

152. A 24 yr old female consulted because of headache palpitations and sweating BP taken 190/110mmHg.to confirm the presence of pheochromocytoma .the preferred drug would be

a. b. c. d.

Piperocan Phenoxybenzamine Histamine Phentolamine

153. Subsequent VMA essay done on the patient above showed elevated results medically, this patient can be managed with the following agents except. a. phentolamine b. propranolol c. atenolol d. phenylephrine e. clonidine 154. The adrenergic receptor most likely responsible for the cardiac stimulation observed ff. intravenous injection of epinephrine is:

c. premature labor d. hypertension 157. A nonselective adrenergic agent, this binds to both beta one and beta receptors and can be used in heart block a. epinephrine b. ephedrine c. isoproterenol d. albuterol 158. Most common adverse effects of beta 2 selective adrenergic drugs a. hypotension b. hemolysis c. tremors d. nausea and vomiting 159. This side effect of inhalational steroids can be minimizes by washing the mouth after its use

a. alpha 1 adrenergic receptors

a. gingival hyperplasia

b. alpha 2 adrenergic receptor

b. oral candidiasis

c. beta 1 adrenergic receptors

c. sore throat

d. beta 2 adrenergic receptors 155. The nonselective beta adrenergic blocking agent that is also competitive antagonist at alpha adrenoreceptors is: a. timolol b. nadolol c. pindolol d. labetalol e. propranolol 156. Ridorine hydrochloride is used in the treatment of: a. parkinsons disease b. bronchial asthma

160. The clinical hallmarks of asthma include the ff. except; a. coughing b. shortness of breath c. fever d. wheezes 161. Considered as controllers in asthma a. beclomethesone b. salbutamol c. ipratropium bromide d. epinephrine 162. True of glucocorticoids, except:

a. decrease formation of cytokines b. inhibit the generation of PGE2 and PGI 2

d. nifedipine 166. Cromolyn sodium as its major action

c. inhibit the production of spasmogens d. reverse the initial bronchoconstriction in asthma 163. This mucolytic agent breaks disulfide bonds and is useful as an antidote for paracetamol a. carbocystiene b. acetylcysteine c. bromhexine 164. Known as chemoattractants, action is on the later phase of asthma: a. LTB4 b. LTC4 c. LTD4 d. all of the above 163. True of salbutamol, except;

a. block of calcium channels in lymphocytes b.

block of mediator release from most cells

c. block of phosphodiesterase in most cells and basophils d. smooth muscle relaxation in the bronchi 169. A 16 yr old patient is in the emergency room. She has a heart rate 130 per min. 0f 40/min and an estimated 1-second forced expiratory volume less than 10% of end rates are audible without a stethoscope. Drugs that can dilate bronchioles during an acute asthmatic attack include all of except; a. epinephrine b. terbutaline c. nedocromil d. theophylline 169. After successful treatment of the acute attack. This patient was on follow-up treatment of her asthma. Succesful strategies currently in use are the following except.

a. given by oral and inhalational route

a. avoidance of antigen exposure

b. maximum effect is within 30 minutes

b. blockade of histamine receptor

c. duration is 4-6 hours

c. blockade of leukotriene

d. blocks muscarinic receptors

d. inhibition of phospholipase

164. These drug cause relatively little respiratory depression unless combine with alcohol a. opiates

170. True of Cromolyn sodium a. in as aerosol that effectively inhibit both antigen and exercise induced asthma

b. benzodiazepine

b. in chronic use slightly reduces the overall level of bronchial reactivity

c. barbiturates

c. efective on airway smooth muscle tone and are effective in reversing bronchospasm

d. anesthetics

d. only a and b

165. Preliminary studies in animals suggest that airway smooth muscle like that in the vascular is effectively relaxed by this agent. Possibly useful in pulmonary hypertension; a. nutric oxide b. cromakallin c. lgE antibodies

180. One effect that theophylline, nitroglycerin, isoproterenol have in common is a. direct stimulation of cardiac contractile force b. tachycardia c. increased gastric acid d. postural hypotension

182. Theophylline acts by; a. inhibiting phosphodiesterase b. inhibiting cylic AMP

b. sulfur dioxide c. nitrogen oxides 188. This is a colorless, irritant gas, generated primarily by the combustion of fossil fuels;

c. inhibiting beta receptor

a. carbon moxide

d. inhibiting muscarine

b. sulfur dioxide

183. The xanthine drugs produce this effects; a. bronchodilation b. decrease gastric and digestive enzymes

c. nitrogen oxides 189. This is a brownish, irritant gas, sometimes associated with fires: a. carbon moxide

c. CNS depression

b. sulfur dioxide

d. all of the above

c. nitrogen oxide

183. Beta adrenergic agents cause; a. stimulation of adenyl cyclase b. inhibition of phosphodiesterase c. antagonize adenyl cyclase d. all of the above 184. Plasma concentration of theophylline is increased when given with a. cimetidine b. rifampin c. phenytoin 185. To relieve the bronchospasm in bronchial asthma, you may give the ff., except a. metoprolol b. ipratropium bromide c. isoproterenol 186. True of ipratropium bromide except a. binds to all muscarinic receptor b. inhibits acetylcholine mediated bronchospasm 187. This is a colorless, tasteless, odorless and non- irritating gas, a byproduct of incomplete… a. carbon monoxide

189. This is a bluish irritant gas that occurs normally in the earth’s atmosphere, when absorbent of ultraviolet light; a. carbon monoxide b. sulfur dioxide c. nitrogen oxides d. ozone 190. Asthma is often exacerbated in patients exposed to this reducing agent when…air as low as 1-2 ppm. It is formed mainly form combustion agent of fossil fuels; a. sulfur dioxide b. rotenone c. carbon monoxide 101. Acute exposure to this alphatic hydrocarbon solvent causes CNS depression, this has led to impairment of memory and peripheral neuropathy: a. tetrachloroethylene b. toluene c. paraqual 102. This compound is a potential environmental hazard that is formed as a contaminant in the manufacture of herbicides; a. DDT

b. dioxin c. aldicarb d. benzene 193. Bone marrow cell in early stage of their development appear to be most sensitive to this drug and can cause pancytoponia and aplastic anemia a. carbon monoxide b. carbon dioxide c. benzene 194. Decrease incidence of EPS side effects; a. chlorpromazine b. triflupromazine c. fluphenazine d. haloperidol e. thioridazine 195. Increase EPS but decrease tendency to produce sedation of autonomic side effects; a. fluphenazine b. chlorpromazine c. thioridazine d. mesoridazine e. piperocetazine True of the pharmacokinetics of antipsychotic drugs;

a. b. c. d.

Highly lipid-soluble and protein bound Longer clinical duration than the half life Prolonged half life A and b

Antagonism of dopamine in the nigrostriatal system

a. b. c. d.

Parkinsonism Galactorrhea Amenorrhea Hyperprolactinemia

e. Psychosis Blocking of dopamine’s tonic inhibitory effect in prolactin release in the pituitary gland causes

a. b. c. d. e.

Parkinsonism Galactorrhea Amenorrhea Hyperprolactinemia Psychosis

Blockade of the dopamine in the mesolimbic and mesofrontal system;

a. b. c. d. e.

Antipsychotic effect Depression Hyperprolactinemia Amenorrhea Galactorrhea

Effects on the medullary periventricular pathway

a. Eating disorder b. Infertility True about clindamycin except

a. b. c. d.

Excellent oral absorption Associated with antibiotic associated enterocolitis Poor bone penetration Prophylaxis of endocarditis in patients with valvular hearth disease for surgical procedures e. Blocks translocation reaction Route of elimination of streptogramins

a. b. c. d.

Fecal 20% urine Bile A and b

Its unique binding site results in no cross resistance with other drug classes

a. Clindamycin b. Erythromycin c. Linezolid

d. Synercid e. Lyncomycin Contraindications for fluoroquinolone use except

a. b. c. d. e.

Elderly A 30yr old patient with complicated urinary tract infection Patients who are in their prepubertal stage Patients taking theophyline A and b

The following are systemic acting quinolone except

a. Ciprofloxacin b. Nalidixic acid c. Gatfloxacin Inhibits the reabsorption of uric acid by a weak acid carrier mechanism at the part of nephron;

a. b. c. d.

Probenecid Coichicine Sulfinpyrazone Allopurinol

Indications for allopurinol use except;

a. b. c. d. e.

Chronic tophaceous gout Allergic reactions to uricosuric agents Recurrent renal stones Normal serum urate Renal functional impairment

True about Aspirin;

a. b. c. d. e.

Poor inhibitor of cyclooxygenase in peripheral tissues Contraindicated in children Crosses BBB and placental barrier by active transport No anti-inflammatory effect All of the above

True about the toxic effects of Aspirin except;

a. Reye syndrome b. Salt and water retention

c. Prolongation of bleeding time d. Antidote is N-acetylcysteine e. Allergic reaction Therapeutic indications of diclofenac;

a. b. c. d. e.

Rheumatoid arthritis Dysmenorrheal Acute MS injury Postoperative pain All of the above

Chemotherapeutic agents classified as plant alkaloid except;

a. b. c. d.

Paclitaxel Teniposide Vincristine NOTA

Chemotherapeutic agent classi fied as podophyflin alkaloid except

a. b. c. d.

Teniposide Vp-16 Vincristine Etoposide

Chemotherapeutic agent classified as alkyiating agent except;

a. Busulfan b. Plicamycin c. Cyclophosphamide Site of alkylation within DNA of alkytating agents is

a. N7 of guanine b. N2 of adenine c. N2 of cytosine Mechanism of resistance to alkytating agents involves;

a. b. c. d.

Increased capability to repair DNA lesions Decreased permeability of cell to the alkylating drug Increased production of glutathione All of the above

Alkylating agent mostly used for CML

a. Busulfan b. Cyclophosphamide c. Dactinomycin Function by cross linking through alkylation of DNA

a. Nitrosoureas b. Antibiotics c. Antimetabolites Cancer chemotherapeutic agent classified as plant alkaloid

a. Vincristine b. Dactinomycin c. Bleomycin Intermediate acting tetracyclines;

a. b. c. d. e.

Demaclocycline Methecycline Oxytetracycline Chlortetracycline A&B

Toxic effects of tetracycline except;

a. b. c. d. e.

Renal toxicity Temporary discoloration of teeth in children GI distress Inhibits bone growth Teratogenic

Toxic effects of chloramphenicol except;

a. b. c. d. e.

GIT disturbances Vaginal candidiasis Reversible aplastic anemia (irreversible) Dose dependent bone marrow inhibition Gray baby syndrome

True about the macrolides

a. b. c. d. e.

Binds to the P sites of the 30 S bacterial ribosomal subunit Erythromycin is the prototype agent Not active against Chlamydia Only inhibitory in action Consist of a single membered lactone ring

Oral preparations of erythromycin;

a. Lactobionate b. Estolate salts c. Glucaptate Which of the ff. is used for prophylaxis treatment of DVT;

a. Heparin b. Warparin c. Aspirin Drug that reverses quickly the effect of oral anti-coagulant drugs;

a. Dicoumarol b. Vitamin K c. Protamine sulfate The ff. drugs reduces platelet aggregation by inhibiting the ADP pathway

a. Clopidogrel b. Abciximab c. Aspirin Account for the most anti-coagulant effect of heparin except

a. b. c. d.

Inhibition of thrombin Inhibition of factor VII Inhibition of Xa All of the above

The presence of tissue thromboplastin is needed to activate this pathway

a. b. c. d.

Intrinsic pathway Common pathway Extrinsic pathway All of the above

Fibrinolytic activators;

a. b. c. d.

Urokinase Plasmin Ticlopidine Vit.k

Which of the ff. clotting factors is greatly affected by heparin except;

a. Thrombin b. IXa c. XIIa Clinical effects of heparin is best monitored by;

a. b. c. d. e.

Prothrombin time Complete blood count Bleeding time Platelet count Partial thromboplastin time

The following are available Vit B12 for therapeutic, EXCEPT

a. b. c. d. e.

Adenosylcobalamin Methylcobalamin (pero ito yung sagot sa samplex) Cyanocobalamin Hydroxocobalamin None of the above

A patient comes into a emergency room is pulse of 140 & a bp of 190/120 he is ….complaining of insomnia. Which of the ff. drugs could have caused these symptoms?

a. b. c. d.

Metaramiol Ephedrine Phenylephrine Amphetamine

All of the ff. are true of trimethaphan except

a. b. c. d.

Increases BP Short-acting Binds to the cholinergic nicotinic receptor Competes for acetylcholine receptor sites

All of the ff. are true concerning scopolamine except;

a. b. c. d.

Plant alkaloid Effects similar to atropine Used in treatment of motion sickness Competes with adrenergic receptors

The toxicity of irreversible cholinesterase inhibitors is treated with

a. Atropine b. Atropine and pralidoxime c. Praidoxime Direct acting cholinomimetics that bind and activate both the muscarinic and nicotinic receptors

a. b. c. d.

Physostigmine Edrophonium Metacholine Malathione

True of physostigmine

a. b. c. d.

Contains a tertiary ammonium group Water soluble Does not enter the CNS Short acting

Which of the ff. antimuscarinic drugs is used by inhalation in the treatment of bronchoconstriction

a. Dicyclomine HCI b. Methscopotamine bromide c. Ipratroium bromide The cholinesterase inhibitor that is used in the diagnosis of myasthenia gravis is

a. Edrophonium chloride b. Ambenenium c. Malathion Atropine and scopolamine will block all the effects of acetylcholine listed below except

a. Bradycardia b. Salivary secretion c. Bronchoconstriction

d. Skeletal muscle??? The efferent nerves of this system supplies only the skeletal muscles

a. b. c. d.

Somatic nervous system Autonomics nervous system Central nervous system All of the above

Which of the ff. statements is true about angiotensin receptor blockers

a. b. c. d. e.

Inhibits angiotensin action more completely than ACE inhibitors No effect on bradykinin metabolism Have the same side effect as ACE inhibitors except for cough and All of the above A and b only

Drug that have been shown to reduce morbidity and mortality in hypertension

a. b. c. d. e.

Diuretics Beta blockers Alpha blockers All of the above A and B only

Antihypertensive agent that is useful in patients with angina;

a. b. c. d.

Beta blocker Calcium channel blocker Hydralazine A and b only

Antihypertensive agents that is useful in patients who also have congestion

a. b. c. d.

Diuretics ACE inhibitors Alpha blockers A and b only

Side effect related to use of ACE inhibitors include which of the ff.

a. Acute renal failure b. Cough and angioedema c. Hyperkalemia

d. A and b only e. All of the above Vasodilator therapy leads to which of the ff. compensatory response

a. b. c. d. e.

Increase renin release Tachycardia Orthostatic hypotension All of the above A and B only

Drugs with positive inotropic action includes which of the ff.

a. b. c. d. e.

Captopril Labetalol Milrinone All of the above A and b only

Which of the ff. statements is true about digitalis glycosides

a. b. c. d. e.

Inhibits Na-KATPase Increases calcium concentration leading to grater systolic contraction Causes increase in intracellular Na concentration which leads to…. All of the above A and b only

Edema in patients with congestive heart failure is due to which mechanism

a. Decrease in renal blood flow activating the RAAS system causing retention of salt b. Increase in venous pressure causes an increase in capillary filling pressure c. Decrease in blood pressure activating the adrenergic system leading to increase arteriolar resistance and systemic blood pressure d. All of the above e. A and b only Beneficial effects of digitalis include which of the ff.;

a. Increase in heart rate b. Increase in cardiac output

c. Increase in production.. d. All of the above

View more...

Comments

Copyright ©2017 KUPDF Inc.
SUPPORT KUPDF